AMA | August 2021

Welcome to the August 2021 Ask Me Anything episode of Mindscape! These monthly excursions are funded by Patreon supporters (who are also the ones asking the questions). I take the large number of questions asked by Patreons, whittle them down to a more manageable size — based primarily on whether I have anything interesting to say about them, not whether the questions themselves are good — and sometimes group them together if they are about a similar topic. Enjoy!

Support Mindscape on Patreon.

[accordion clicktoclose=”true”][accordion-item tag=”p” state=closed title=”Click to Show AMA Questions Only”]Click above to close.

Arthur C. Quark
In *The Big Picture* you describe the Fine-Tuning-Argument (FTA) like this:
In Bayesian language, the likelihood of LIFE APPEARING in the universe might be large under THEISM, and small under NATURALISM. We can therefore conclude that OUR VERY EXISTENCE is strong evidence in favor of THEISM.
In addition to everything you mention to decrement its credibility, the argument itself *feels* wrong. If I mad-lib the argument, I can prove that I’m psychic (or that bear taxes make bears go away, or the TSA makes terrorism stop).
Is the argument itself a logical fallacy?
In Bayesian language, the likelihood of ME WINNING THE LOTTERY might be large under I’M PSYCHIC, and small under I’M NOT PSYCHIC. We can therefore conclude that MY WINNING THE LOTTERY is strong evidence in favor of ME BEING PSYCHIC.

Brendan Hall
Does general relativity become greatly different when one analyzes say 4 spatial dimensions(to make 5d spacetime) as opposed to 3?

Dave Williams
PRIORITY QUESTION
I recently posed myself a question “Suppose I journeyed 1000km in 10hrs but on arrival I was only doing 40km/hr. Those that greeted me only knew that I traveled 1000km and that I arrived 40km/hr. Hence they thought I took 25hrs when I actually took 10”. Such a trip mimics our observations of the Universe. I easily solved the dilemma of conflicting journey times by assuming that the product of the distance [40km] and the measure of time [1hr] was a constant. Translating this idea to the Universe and interpreting our observations in this manner solves all of the problems: inflation, dark matter, dark energy, duality of light etc.
My question is: ‘Re-formulating our current theory of the big bang but replacing the constant speed of light, i.e. distance/time with a constant distance*time, solves all of the major obstacles of the theory whilst supporting its many successes.’ Has such a consideration ever been contemplated? And if so, on what grounds does it fail to satisfy?

Justin Bailey
Is there a minimum speed of light at which the universe is still “interesting” (i.e., has beings like us)? What would that universe be like?

Boglarka K
Is dark matter discussed or framed as a limit of our knowledge in physics?

Jonny
At a base level are photons much different than any other matter particle? Apologies for the basic question here 🙂

Carlos Nunez
Who is your favorite superhero and why?

Sam Buck
Careful viewers of the Veritasium YouTube channel may have noticed a cameo from you in a recent video, featuring a bet between Derek and Professor Alex Kusenko, although sadly we neither saw nor heard much from you during the event. Can you talk a bit about your experience being a witness to the contest? Were you, like Bill Nye, persuaded by Professor Kusenko’s presentation or surprised by the outcome of the bet?

LINEU D MIZIARA
In your lectures about the Higgs boson in “The Great Courses”,you said that,because the Z and the W particles are originally massless (and acquire mass through the Higgs field),these particles have to be always moving, and that is how Nature knows that they are left-handed.However, isn’t movement relative? What about those particles relative to which the Z and the W are stationary?

Mikolaj Szabo
Is it true that current quantum computers are not the ENIACs or EDVACs of quantum computation but more like Babbage’s Analytical Engine? Or even this is a stretch? What are the chances that quantum computing won’t work in the end, as Wolfram predicted in your interview with him?

Robin van Dijk
My question is about how you come up with ideas for writing a paper? Is it just by talking with other physicists about your work or are there other things that influence what you write papers about?

********************* gravitons
STEW HAYNE
I believe you have said, that we can be a bit loose and describe gravity as a force even though it is really the curvature of spacetime. If so, why do we speak of gravitons as a particle mediating the force of gravity?

Peter Solfest
Are gravitons merely the particle duality of gravitational waves? If not, why is there so much confidence that they exist?

Andrew Jewell
There are four “forces”. With gravity and electromagnetism, there’s a simple formula to calculate force, which one can plug into F=ma; but with the strong and weak forces, there are simply strange phrases such as “is involved in radioactive decay” and never an equation. So what does “force” mean with the strong and weak forces? Is there a way to calculate an “F” which can equal “ma”?
**********************

Felix Roswald
Is there a 1:1 ratio of electrons to protons in the universe; or in other words, is there a net charge to the universe?

Anonymous
I really enjoyed your discussion in the last AMA about the difference between being a public intellectual and being an activist. I thought about it some more, and I’ve realized that the two roles aren’t completely disconnected. An intellectual whose conclusions just aren’t palatable to the average person’s sense of decency won’t be seen as very credible, even if they have a rigorous and consistent thought process. An activist whose conclusions just aren’t realistic won’t be seen as credible either, even if their ideas support a cause. Suppose that an intellectual needs to think like an activist sometimes. Where would you set the boundary between staying in touch with the society we live in on a basic level, and letting personal feelings and wishes get in the way of objectivity?

Kyle Maurer
Why isn’t gravity/the gravitational field constantly “decohering” quantum systems and getting rid of their wavelike natures? Seems like even a single particle would be constantly updating spacetime with its position and energy.

Lester Su
I think you’ve said before that all good Bayesian should update their priors in the same way based on objective new facts. But if a Bayesian starts out with their subjective priors, surely those priors would (and should) also inform how they assess new facts and therefore how that will update those priors? Or is there an objective standard to how you assess new facts as a Bayesian independent of your priors?

Fabian Rosdalen
Regarding your work in very fundamental questions of the universe, have you ever had to deal with any existential anxiety/pain or similar? And do you have any thoughts on how quantum physics could influence these matters, either positively or negatively.

ratboy.exe
I recently listened to your talk with Philip Goff on panpsychism and was absolutely enthralled. One thing that struck me was the discussion on “intrinsic” properties – those that don’t govern observable behavior and aren’t captured by mathematics. It reminded me of your discussion with another guest, Max Tegmark, who seemed to argue that there is no such thing, to the extent that the universe is nothing more than an abstract mathematical structure, and even further, that this universe is not uniquely “real” among mathematical structures.
My question is this: if your position is that there are no intrinsic properties, what in your view distinguishes the world from the abstract mathematical structure described by Tegmark? And if, in contrast to Tegmark’s view, this universe possesses a unique property called “existence”, would that qualify as a sort of intrinsic property that we get to be privy to by virtue of our being internal to the structure?

Ken Wolfe
Back in the day we were all taught that there were just three types of chemical bonds: covalent, ionic and metallic. I recently read an article by Philip Ball about new research which suggests there might be new types of chemical bonds, such as ones that blur the line between covalent and metallic. This got me to wondering whether any new developments in quantum electrodynamics could possibly give us new insight into how chemical bonds might work. Do you think that is possible? A shorter version of the same question: could changes in the Standard Model of Physics change our understanding of how chemical bonds work, or is that pretty much locked in by now?

ryan morrison
I’m not an expert in psychology, but I think Maslow’s hierarchy of needs and the idea that we can make progress as humans to be pretty attractive. Do you ever think about the idea of transcending the self or have you moved away from that kind of thinking? Do you think it’s possible to be totally fulfilled?

Casey Mahone
I have been thinking a lot about your notion of emergent spacetime. In my mind it raises a similar issue as the hard problem of consciousness. In both cases, it seems completely unclear to me how something of a totally different “kind” can arise simply through emergence.

Rob Greyber
This month, I wanted to ask a question from my 7 year old daughter, Liana.
Professor Carroll, if you were standing on the photon sphere of a black hole, and you looked forward, you would see the back of your head. Is that right?
What would happen if you looked up? What would you see? And what would you see around you, to the left and right?

Jesse Rimler
Music emerges from the naturally occuring overtone series, which is the same everywhere in the universe (at least, I assume it is). The scales and chords we build out of this vary from culture to culture. As a thought experiment, I’ve tried to imagine a fundamentally different kind of musical harmony, beyond cultural or timbral differences. Am I right to conclude that it would have to be a universe with a different kind of math, since the octave and the overtone series are based on mathematical relationships?

Michael Edelman
In the last AMA you dismissed the idea of nations having intelligence or consciousness, likening to anthropomorphizing. Given that consciousness appears to be an emergent property of sufficiently large networks of neurons, why shouldn’t we also see consciousness arise from networks of more complex organisms like humans and animals? George Dyson suggested this in his “Darwin Among The Machines” twenty four years ago, drawing parallels to Hobbes’ “Leviathan.”

Peter B
One of your earlier guests, Leonard Susskind, has argued that inflation must create infinite open universes with negative curvature. Yet, you have mentioned that we live in a de sitter universe with positive curvature. I am confused: Are we living on pringle chip or a giant balloon?

Mark Zug
Regarding photons interfering with themselves: what is wrong with the idea that EM energy must travel as waves but only be detected as particles?

Marc Mathieu
Could quarks in a proton run into their virtual antiquark counterpart and exchange places with a virtual quark mediated by a gluon resulting in a virtual quark to replace the original quark.

Michael Ehling
Can you suggest one or two books that give an overview–perhaps with some history–of the philosophy of science and highlight some of the current issues those in the field are tackling?

Connor Scott
If a neutron star is propped up by the Pauli exclusion principle, and if the Pauli exclusion principle only forbids fermions from being in the same quantum state, then wouldn’t it be the natural assumption (in the case of a neutron star gaining mass and collapsing into a black hole) to say that the simplest ground state of the fermions is no longer to distinguish their quantum states by position, but by some other factor?

Chris Shipton
Is there a relationship between the halting problem and the traveling salesman problem?

G. John Dick
Given an Everettian perspective, what would be the consequence if our universe (the vacuum) happened to be in a metastable state with a decay time of (only) a million years. Could we tell?

Anders
The Kalaam Cosmological proof for God is regularly trotted out (“Everything that begins to exist has a cause. The universe began to exist. Therefore the universe has a cause.” And with a little handwaving you get to God). This seems like something a cosmologist would have something to say about. Why don’t you accept it?

Siddhartha
Can any (seemingly) non-deterministic universe be truly random at the fundamentally level? Even if entities inside the universe cannot tell, doesn’t the universe itself somehow need to “pick” one (or all in MWI) possibility following some deterministic process? I know we cannot really demand anything from base reality, but I’m also finding it hard to imagine how base reality itself would operate under uncertainty in what to do “next”.

Corby Ziesman
(Priority Question!)
My question ties into a “quantum suicide” type thought experiment, but related to blockchain security.
Basically your cryptocurrency money is protected by a 256-bit private key, nearly impossible to guess. But one could use a quantum random number generator (or the Universe Splitter app) to generate 256 1-bit quantum results. This means you would have 2^256 timelines in superposition where each version of you tries a unique 256-bit permutation. This exercises all possible cryptocurrency private keys, meaning that there’s a timeline where you successfully guessed every single user’s private key.

Allison
Could quantum networking be used to build a telescope that spans planets?

Stefan Bernegger
I very much liked your conversation with Stephen Wolfram. He has not yet shown that his approach can actually explain our universe but at least so far there is nothing disproving his approach. I am, however, not sure how he will manage to overcome the many orders of magnitude between the scale which is reachable by computations and the scale where observable physics starts.
I am thus curious to hear your main take-aways from this conversation and if it somehow changed your view on how to address the open questions on a most fundamental level.

Brad Malt
In 1968, track and field star Bob Beamon broke the world record for long jump by almost two feet. This is a margin so big that it was named one of the 5 greatest sports moments in the 20th century by Sports Illustrated, and it remains an Olympic record over 50 years later.
Do you think this amazing jump could have been an instance of the wave function collapsing to a very unlikely event—the sports equivalent of putting one’s fist through a solid table? Before you say this is too unlikely to expect it to have happened, consider that the relevant universe of events may not be only long jumps in the summer Olympics, but something much larger like all athletic events in the summer Olympics, or all athletic events in the history of the universe, or all muscle contractions in the history of the universe.

Stephen Barnard
You’re invited to host a panel discussion of the greatest physicists of all time, living or dead, and language is no barrier. But you’re limited to three. Possible choices pre-approved by the committee are:
Aristotle, Galileo, Newton, Einstein, Bohr, Everett, Feynman and so on.
The question is obvious.

Peter Behnam
I know you have a distaste for “it seems” but it seems logically obvious that it HAD to be possible at the big bang for emergent phenomena of high complexity like life and consciousness to evolve though it would “seem” impossible that one could have conceived of such things by just looking around at the raw materials and yet here we are. What are ‘potential’ higher order emergent phenomena that ‘seem’ impossible might we be able to conceive of emerging…eventually…somewhere…somehow?

krathorlucca
What exactly does it mean to be a moral constructivist? Is it really different from being a moral realist? For example, people often object to moral realism by stating that there can be no foundation for moral knowledge to be built up from. However, when it comes to physics and mathematics, people are far more willing to accept we are exploring some objective reality, rather than merely ‘constructing’ it. But in physics or mathematics there can be no foundation for knowledge either! (we could be tricked by a demon etc. and gödels incompleteness theorem)
If we can’t justify our beliefs in areas like math and science more than in morality, then can’t we say that lacking a foundation is not an impediment to realism? Because now there is a dichotomy between moral knowledge on the one hand, and scientific and mathematical knowledge on the other hand, even though they all lack a foundation.

*****************************
Alexander Cordova
How is it that massless particles like photons can have a non-zero momentum? I’ve worked through the “special relativity” chapter of an undergraduate physics textbook and the justification seems to be given by the equation:
E^2 = (mc^2)^2 + (pc)^2.
However, it also gives the following equation for relativistic momentum:
p = mV/sqrt(1-v^2/c^2)
No matter how I rearrange these equations or try to plug in various equivalent values; it would seem as though plugging in a rest mass (m) of 0, would result in momentum (p) also being equal to 0.

Thomas Prunty
Just in case Sean doesn’t answer this: If you put v=c and m=0 in the relativistic expression for momentum you get 0/0, in other words undefined. A photon is a quantum mechanical object, and so you need quantum mechanics to get it’s momentum, relativity doesn’t have an opinion on the matter.
*********************************

Thom Quine
PRIORITY QUESTION: You wrote in “From Eternity to Here” that “Entropy, quite literally, makes life possible.” I have an inkling that entropy is the motor of evolution, the driving force that requires things to either persist or to perish. Can you make a hypothetical argument for why this might be true, and a parallel argument for why it might be the wrong way to look at things?

douglas albrecht
infinity and infinitesimals seem to create challenges in physics and it seems that they requires what seem like “work arounds”. Energies at infinitely small distances require cut offs. Cosmology seems to blow up the beginning of time with inflation to explain certain otherwise unexplainable. Replacing point particles with vibrating strings is another type work around to explain problems of reducing things like charge to an infinite point. Quantizing space time also seems another way of dealing with this. So I wonder if maybe it’s the math that needs evolution to deal with limits or if we are just not equipped to deal with this extreme points.

*************************
Matt Hickman
My question is about the black hole information paradox.
From my understanding, one of the guiding principles used to think about this problem in physics was that “information is not destroyed in quantum mechanics.” If I were a believer in the Copenhagen interpretation, I might say that information is destroyed all the time! A wavefunction happily evolves according to the Schronidnger equation and them when you measure it, collapse, eigenstate, information gone.

Humberto Nanni
may you please comment on what is lost if unitarity is loss? Like, which parts of the current theories will be affected?

Rich Hogg
Hawking radiation is usually described as emerging in a specific thermally randomized configuration, but under the MWI, wouldn’t we more correctly think of it as emerging in a superposition of all possible configurations, which then decoheres into specific configurations in separate branches?
*************************

Daemonhat
Would you rather have legs as long as your fingers or fingers as long as your legs(fully functioning) and why?

Paul Hess
What is your method to choose which papers to read, out of the near infinite deluge?
Do you have several passes where your first skim many papers, then from that set choose a much smaller subset to re-read more closely? Or do you read papers that others are talking about in your circles?

Sam Cox
I wonder if the missing antimatter problem — the Baryogenesis Anomaly — is somehow connected to the mysteries of dark matter and dark energy. Perhaps the missing antimatter was “sponged up” by or morphed into dark matter at an early moment after the Big Bang. Also, is it reasonable to expect there exists dark antimatter?

andrew vernon smith phd
Can you explain whether or not it is possible that the weak force may have been involved somehow in bringing about the universe, including time and space, via the big bang or otherwise, and if so, whether it is possible that the right-handed matter and left-handed antimatter may have proceeded backward in time (since only left-handed matter and right-handed antimatter interact with the weak force in our forward-in-time reality?

Tony B
This question is inspired by the recent movie Tenet, which engages with ideas on the entropic arrow of time more than many. You have spoken many times about how we remember the past because in that direction, our extrapolations are pinned down by the low entropy boundary condition of the beginning of the universe. If the low entropy boundary condition were in the other direction, we would remember in that direction in exactly the same way. Do you have any intuition as to what kinds of events could happen at the boundary between two regions of a universe, one region having a low entropy boundary condition in one direction and the other having a low entropy boundary condition in the other?

jeff babon
I’m trying to understand how much of the twin paradox is due to the fact that one twin accelerates while the other doesn’t. So, if I have a circular track approximately 1 light year in radius and I’m travelling around the track at 0.9c then I will feel a centripetal acceleration of approximately 1g. If this track is next to earth then as I pass it my twin brother and I will synchronise our watches. By the time I come around again I think I will have aged more slowly to my twin even though we have both been feeling 1g of acceleration the entire time. If that is true what makes my frame of reference special, is it because only I have moved relative to the universe as a whole?

Jeff B
You have stated that you are against the idea of a teleology underlying physics. However, it seems as though this would be difficult to judge. After all, the laws of physics may be progressing towards some final teleological state, but the path that it takes is simply too complex for us to notice or understand. In fact, this seems logical based on the fact that matter tends toward the lowest energy state. This is reminiscent of the ancient notion of elements “wanting” to rise or fall. What do you make of this?

George Atanasoff
Are you concerned about carbon emission? Now we began using electric car transportation. The battery manufacturing is very carbon-intensive, but the subsequent battery recycling or disposal will create a nightmare 20 years from now.

Chris
Most of your discussions of philosophy that I’ve heard seem to be within the Western framework, what are your thoughts on others such as Eastern philosophy? This relates to the “topic” of metaphilosophy – any easily summarisable views on that?

***************************************
Sandro Stucki
In the podcast with Elizabeth Strychalski the idea that cells are information processors and climb entropy gradients came up again. But how is the idea that physical/biological/computational systems process information and make “choices” or use “strategies” to “optimize” certain outcomes compatible with a purely deterministic evolution of the universe? In what sense can an organism or self-replicating piece of chemistry influence its future? That kinda presupposes that there were alternative futures to begin with; that information about the environment can truly be exploited.

Jason
You’ve said before in answering questions about free will vs determinism, that as a compatiblist, you find the interesting question to be about how to operationalize that. So how do you operationalize that? When does it make sense to treat people as if they have free will and when does it not?
*******************************************

Preston Justice
You’ve been quite successful dabbling in philosophy, therefore, is it logical or realistic to assume one could go counter clockwise and find their way into contributing to the sciences without “shutting up and calculating” from the field of philosophy of science/physics in the humanities department?

David Wright
The latest podcast with Elizabeth Strychalski on Synthetic cells was fascinating and touched on some of the same exciting questions about the boundary between physics, chemistry, and biology that Michael Levin is dealing with in his work on morphogenesis at Tufts.
You ended up discussing whether living systems actually require a “cell membrane” and if it was possible that there was a more general synthetic form for biological systems. This reminded me of one of your earlier guests, Karl Friston, who has identified a model for biological systems based on his Free Energy principle. The model identifies internal states, external states, active states, and sensory states that operate to minimize the difference between a known internal state and Bayesian predicted external state. A cell membrane may simply be an evolved way to limit the computational bandwidth needed to process the model algorithm by reducing the number of internal state variables. What the cell model and non-cell model share in common is that they would both be based on Markov blankets with different scales and numbers of states.
Do you think Dr. Strychalski should consider Friston’s model in her research?

Brendan
I was interested in reading your textbook, “Spacetime and Geometry” but was wondering how much prior knowledge in Mathematics and Physics is roughly required. I have all your other books and have followed along to your Biggest Ideas in the Universe series, but was unsure whether that would still be enough without a rigorous understanding of the underlying math and physics? My background is in Computer Science and it has been over ten years since I took a differential equations class.

Richard Graff
Try as I might I can’t grasp the connection between symmetry and conservation laws as worked out in Noether’s theorem. Can you explain the relationship in relatively non-technical terms, or is this one of those somethings deeply hidden that requires an understanding of the math to comprehend?

Rebecca Lashua
Do you agree with Tegmark’s basic premise that the universe is essentially mathematical in nature? Certainly a complete theory of everything would be written in mathematics, but is there something that “breathes fire” into the equations, or is the universe just the equations (or somehow isomorphic to them)?

Anonymous
I have heard physicists say that time and space switch roles inside of a black hole. I also have heard them say that the whole universe is inside a black hole. Are these statements true in some sense?

ashik dragneel
What’s is Universality of Computation? Are Humans universal too i.e. can humans understand anything and everything in the universe given our limited perception of the world?

Varun Narasimhachar
Why do you think our understanding of quantum mechanics is incomplete? I feel we understand it operationally just as well as we do classical mechanics, but seem to insist on fitting it to our classical intuition by adding unnecessary features beyond what’s operationally accessible. As for questions like “What really exists?”, I think we had no meaningful answers in classical mechanics either (e.g. Does mass exist? Do electric fields?).

Kathi Seeger
In your recent paper “Consciousness and the Laws Of Physics” you elaborated on the Core Theory. You’ve also made it very clear what its domain of applicability is.
Since Frank Wilczek proposed the Core Theory in 2015 and you endorsed it ever since:
What’s your impression, did it gain more attention and approval in the physics community over time?
Did Frank Wilczek approve of the way you constructed the Core Theory equation?

Samuel Benjamin
You’ve mentioned many times that the words we use in modern physics are poor descriptors of the phenomena we observe (“wave function” being a particularly bad one, although I still have to think hard about the various uses of the word “space”).
Have you ever come up with alternatives for some of the more challenging terms and phrases, that you feel do a better job at helping others understand the concepts? Do you know of any other languages that describe these concepts “better” than English?

David de Kloet
How do you decide when to end a podcast episode? Are guests ever caught by surprise when you suddenly say “thanks for being on the mindscape podcast” or do you prepare them in some way?
[/accordion-item][/accordion]

[accordion clicktoclose=”true”][accordion-item tag=”p” state=closed title=”Click to Show Episode Transcript”]Click above to close.

0:00:00.0 Sean Carroll: Hello everyone, welcome to the Mindscape podcast, the August 2021 Ask Me Anything episode. I’m your host, Sean Carroll. You know how this works probably by now, it’s an Ask Me Anything, so people write questions and I do my best to answer them. The people who get to ask the questions are Patreon supporters, so if you want to be part of the question-asking brigade, just join up at patreon.com/SeanMCarroll, not that expensive, and you get not only the right to ask questions every month for the AMAs, but also you get ad-free versions of the podcast and a feeling of well-being for supporting something good and being a member of the community.

0:00:39.6 SC: Now, as we’ve said before, the number of questions has grown fairly large, so we cannot answer all of them, but I go through and I try to figure out which ones I have interesting things to say about. So again, if you don’t get your question answered, not necessarily ’cause your question wasn’t good, it just didn’t vibe or resonate with what I had to say in this particular day. So I hope everyone’s doing well, we’re in the middle of the summer, post 2020.

0:01:05.1 SC: So the first summer that should be post-pandemic, but we’re not post-pandemic because many people have not gotten vaccinated yet, and therefore we’re still in the middle of a pandemic. And I hope that it doesn’t get worse, but as long as we haven’t vaccinated the whole world there’s still plenty of opportunities for the virus to mutate, we already have the Delta variant and other things. And rather than just getting rid of this virus, we’re sort of letting it flit from person to person because we don’t want to as a society take the necessary steps to fix it, this makes me very sad.

0:01:36.2 SC: But nevertheless, several months ago, I decided I was just going to be optimistic about this, and therefore I made travel plans. So tomorrow, as I record this AMA episode, it will be the first time I go to the airport to step on an airplane in roughly a year-and-a-half. So we’ll see how that goes. I’m going to Santa Fe to go to the Santa Fe Institute where I am now officially a fractal faculty member, that’s the big news of this month in my life. Some of you know that I am an external professor, have been an external professor at Santa Fe.

0:02:08.1 SC: Actually, some of you might not even what know the Santa Fe Institute is, it’s a research institute in Santa Fe dedicated to the study of complex systems, which is an area I’m very much getting into these days. And so most of the people who are involved with Santa Fe in some way or another actually don’t live and work there, they live and work somewhere else, but they visit there regularly. And so there is a category called external professors, where in return for being able to say, “I am affiliated with the Santa Fe Institute,” you promise to do something for them, like help them choose postdocs or organize a conference or spend some time there or whatever it is.

0:02:47.2 SC: So I’ve been there for a couple of years. I didn’t get really to take advantage of it ’cause you’ve we’ve had a pandemic for the last year-and-a-half. But now this upgrade to fractal faculty means that it’s just more of an obligation. So I will go several weeks a year, at least six weeks every year, spend time in Santa Fe and really take part in what goes on there. I’m going into details here, ’cause someone asked a question I didn’t include in the AMA, but here’s the answer. “What does it mean to be an external professor at the Santa Fe Institute?” It just means you do research that is involved with Santa Fe, you spend time there, you talk to people, I’ve already been on many Zoom meetings, etcetera, with people at Santa Fe, even though I haven’t been physically there.

0:03:28.0 SC: And I’m very proud to be one of the first two fractal faculty members in existence, the other one is Melanie Mitchell, former Mindscape podcast guest, an expert in artificial intelligence and computing more generally, she was, you remember, Douglas Hofstadter’s graduate student back in the day. So Melanie and I are the first two fractal faculty, and it’ll be fun, it’s one of those places, SFI, where every time I go, I am just energized with all the ideas that are bouncing around. ‘Cause some people are thinking about physics and thermodynamics, some other people are thinking about economics, other people are thinking about linguistics or the origin of life, and it’s just a kid in a candy store feeling for myself.

0:04:07.2 SC: So that will be for two weeks, I’m going to go to Santa Fe and then I’m going to come back to LA for two weeks, and then I’m spending the fall on sabbatical at Harvard as a visitor in the Harvard Philosophy department. I’m sure I’m going to also talk to some physicists while I’m there, I have many friends in the Physics department and Astronomy department at Harvard. But I want… I was a grad student there, but I want to really start thinking more deeply about questions of emergence and causation and the nature of the laws of physics, Humeanism and things like that. So I thought that that would be a wonderful place to do that, and of course not only is Harvard a very good place, but it’s near other good places, so I can talk to a whole bunch of people.

0:04:48.6 SC: They have an official visitors program, and I applied and I was invited to go do that, so that’s where I’ll be for September, October, November. So that’s also another travel thing, I hope it goes well, if I end up sitting in an apartment in Boston and just Zooming with people at Harvard, that won’t be fun. So please get vaccinated, please get yourself some immunity, much, much greater immunity to the effects of this terrible disease, so that we can get over the pandemic and return to normal life. And with that, let’s go.

[music]

0:05:34.3 SC: Arthur C. Clark, probably not their real name, says: In The Big Picture, you describe the fine-tuning argument like this. In Bayesian language, the likelihood of life appearing in the universe might be larger under theism than under naturalism. Then we can therefore conclude that our very existence is strong evidence in favor of theism. In addition to everything you mentioned to decrement its credibility, the argument itself feels wrong. If I mathlib the argument, I can prove that I’m psychic or that bear taxes makes bears go away. Is the argument itself a logical fallacy? In Bayesian language, the likelihood of me winning the lottery might be large under I’m psychic and small under I’m not psychic. We can therefore conclude that my winning the lottery is strong evidence in favor of me being psychic.

0:06:17.8 SC: So the answer to this is, you’re not quite being a very good Bayesian yet, Arthur, sorry about that. This is how Bayesians reason. So for one thing, don’t use the word prove. You said, “I can prove I’m psychic.” No, you can’t prove you’re psychic, but you can gather evidence that you’re psychic and you can gather evidence against that you’re psychic. And you count it all up. Okay. The point of me making this argument was that… It was not to show the fine-tuning argument works, ’cause I don’t think it does work. It’s to show that if that’s all you knew about the universe was that life existed, then you could count that as evidence for the existence of God, because there is a likelihood function, and I don’t think that it’s unreasonable to think that the likelihood of life under theism is larger than the likelihood of life under naturalism. That’s completely reasonable.

0:07:07.5 SC: And you just have to take the argument seriously, you have to take Bayesian analysis seriously. So for one thing, there’s a prior, of course, if your prior is very small for one possibility or the other, that will swamp any one piece of evidence that you have. But more importantly, there’s other evidence out there, and that’s what really matters. A good Bayesian will admit that there are certain facts about the world that favor even the wrong hypotheses, right?

0:07:36.4 SC: If you think about all the hypotheses you can make about the world and then focus in on the subset where some people actually believe these hypotheses, it’s overwhelmingly likely that there are certain aspects of the world that are better explained by these wrong hypotheses than by the right ones, a priori, right, because otherwise, no one would believe the wrong hypotheses. But to be a good Bayesian you have to take all of the evidence into account, and this is the mistake that is made by people who put forward the fine-tuning argument in my mind, is that they focus in on one fact that there is life in the universe, and they say the likelihood of life under theism is greater than likelihood of life under atheism/

0:08:16.7 SC: And my point is, if you want to play that game, which is a perfectly legitimate game to play, you have to look at other aspects of the universe and say, Are those other aspects of the universe more likely under the theism or atheism? And the reason why they don’t want to do that is that over and over again, the aspects that we observe about the universe are way more likely under naturalism than under theism. So when you say the likelihood of me winning lottery might be large under I’m psychic and small under I’m not psychic, we can therefore conclude that my winning the lottery is strong evidence in favor of me being psychic.

0:08:52.1 SC: Yes, it is a pretty strong evidence in favor of you being psychic. Now, there might be plenty of other evidence that you are not psychic, but think of it this way, what if you just thought of what the right lottery number was going to be and played it and won five times in a row? You might say, Well, hey, actually that is pretty good evidence that I’m psychic, ’cause if I can consistently win a billion-to-one odds lottery, maybe there is something going on there, right? But if you can, if that’s true, when you do it five times in a row or 10 times in a row, then there’s some evidence in that favor, even if you only do it once. That evidence need not be so good that you believe the hypothesis, a good Bayesian never puts their credence to zero or one for any reasonable hypothesis.

0:09:35.6 SC: So it moves your credence whatever amount, but that doesn’t mean that you’re done, that doesn’t mean that you can stop thinking. I think it’s very, very important to really take seriously the fact that you can’t play the Bayesian game only on the evidence you like, you have to play it on all of the evidence, that is the point of that argument I was making in The Big Picture.

0:09:56.3 SC: Brendan Hall says: Does general relativity become greatly different when one analyzes, say, four spatial dimensions to make 5D spacetime as opposed to three?

0:10:04.3 SC: So the short answer is no, it’s not really all that different, general relativity in higher numbers of dimensions, but the reason why it’s an interesting question is because, interestingly, general relativity is very different in two spatial dimensions, so if you have three-dimensional spacetime, there is kind of a phase transition. One-dimensional general relativity or two-dimensional general relativity are very different than general relativity in four or five, six, etcetera, dimensions. Is this some kind of anthropic argument that we need at least three spatial dimensions to make the world interesting? Probably not, but I’m just throwing it out there for you.

0:10:37.6 SC: And actually, let me just take advantage of this opportunity to go into a little bit more technical detail than I would usually go into here, ’cause it is a very interesting subject. What is the way in which… And in what sense is general relativity different in two-dimensional space, so three-dimensional spacetime rather than the real world. Well, the… And I can’t help but introduce the jargon here, sorry about that. We often say that general relativity says that matter and energy in the universe drives the curvature of spacetime.

0:11:04.1 SC: That’s probably what you’ve heard. You’ve heard me say it, if no one else, but there is, it turns out there are different types of curvature in spacetime, and this is just a mathematical fact, and I’m not really going to explain the different types, but there are two different types, and one is called the Ricci curvature, named after Professor Ricci, who’s an Italian geometer, and the other is called the Weyl curvature, named after Herman Weyl, who was a German mathematician. Weyl, W-E-Y-L, okay.

0:11:33.2 SC: So there are these two different types of curvature and they’re both bundled up into the Riemann curvature. The Riemann tensor is the geometric object in general relativity that tells you all of the curvature, and different pieces of the Riemann tensor correspond to the Weyl curvature or the Ricci curvature. And the point is that Einstein’s equation, which relates the curvature of spacetime to matter and energy, says two things. It says that the Ricci curvature of spacetime, that part of the curvature, is exactly determined by the distribution of energy and momentum. Okay, so if you tell me what the energy is, I constantly know what the Ricci curvature is, that’s a geometric… I should say an algebraic relation that is given to us by Einstein’s equation of general relativity, tell me the energy, I’ll tell you the Ricci curvature.

0:12:19.4 SC: Whereas the Weyl curvature is not completely fixed by the amount of energy and momentum. In fact, what Einstein’s equations tell you is that there is an equation that relates the evolution of the Weyl curvature, but it doesn’t tell you what the initial conditions are, so that’s crucially important for general relativity, because the Weyl curvature characterizes things like gravitational waves wave. When you have a gravitational wave propagating in empty space, if I just tell you, Well, space is empty, there’s no energy and momentum there, that doesn’t tell you whether there’s a gravitational wave there or not, right. Telling you what the energy momentum is in some part of the universe has nothing to do with whether or not there’s a gravitational wave. If it did then there couldn’t be any gravitational waves, right? Because in empty space, there wouldn’t be anything to source them, but the gravitational waves are Weyl curvature, which are not fixed, their evolution is governed, but the initial conditions are not fixed by Einstein’s equation.

0:13:20.7 SC: So why am I telling you all this? Well, because the way the math works out, there is no such thing as Weyl curvature in three-dimensional spacetime, it just goes away, so zero, identically there is only Ricci curvature. So in three-dimensional spacetime, there are no gravitational waves, there is no curvature of space outside matter, right. So it becomes a little bit more interesting to think about how gravity would work in three-dimensional spacetime, and in fact, I’ve written papers about it. If you have a cosmic string, which is infinitely long and perfectly straight, then along the direction in which the cosmic string is pointing, you can just ignore everything that happens, there’s a symmetry with respect to boosts and translations in that direction. So a set of cosmic strings that are infinitely long, perfectly straight and parallel to each other are equivalent to point particles in one lower dimensional spacetime.

0:14:18.0 SC: You just ignore the dimension along the strings pointing. So you can actually analyze questions of cosmic strings in a two plus one-dimensional, three-dimensional that is, space-time context. And that makes things easier. And the reason why that’s interesting is because Richard Gott of Princeton pointed out that you can build closed time-like curves using infinitely long straight cosmic strings. So Alan Guth and Eddy Farhi and Ken Olum and I analyzed that question using this two plus one-dimensional point particle version of the problem, the spherical Cow version of the problem, and we show that if you had a time machine built into the universe, it would stay there, but you can’t build it if it’s not there to start. Gerard ‘t Hooft also wrote some things along those lines. So anyway, all of which is to say, gravity is different in two or three-dimensional spacetime, four or more is more or less the same up to some details.

0:15:10.7 SC: Dave Williams says: Priority question. I recently… Oh, remember that’s right. So priority questions are, I said that everyone gets one priority question in their lifetime, and it’s an honor system here, I’m not keeping track of when asks these questions, but… And you get to ask one priority question as a Patreon supporter that I promise I will answer. I don’t promise that I will give you an answer that you would like or will satisfy you, but I’ll promise I will answer here in the AMA. So Dave Williams says: I recently posed myself a question. Suppose I journeyed 1000 kilometers in 10 hours, but on arrival, I was only doing 40 kilometers an hour. Those that greeted me only knew I’d traveled 1000 kilometers and I arrived 40 kilometers an hour. Hence, they thought, I took 25 hours when actually I took 10. Such a trip mimics our observations of the universe. I easily solve the dilemma of conflicting journey times by assuming that the product of the distance and the time was a constant. Translating this idea to the universe and interpreting our observations in this matter solves all of the problems, inflation, dark matter, etcetera. Has anyone ever thought of this is roughly the question.

0:16:16.7 SC: So the short strategy, the simple strategy you should use in these situations is to think to yourself, are decades of very smart professional scientists complete dummies or not? And the answer is no, they are not complete dummies. So this assumption that the expansion of the universe is just constant or you can just extrapolate what you see in your local neighborhood, no one ever made that assumption, that’ll be a dopey assumption to make. What we do is two things, number one, we predict the evolution of the expansion rate of the universe using Einstein’s equation of general relativity, and then we test those predictions. So we test those predictions with the microwave background, with redshifts, with the nucleosynthesis in the very, very early universe, and there’s a relationship between the density of the universe and its expansion rate that is predicted by Einstein, and so far that relationship fits all of the experimental data, it passes all the tests.

0:17:11.3 SC: And the other thing from a model, independent theory independent point of view is we just measure it, so that’s how the acceleration of the universe was measured, it was not by looking at the expansion rate of the universe here in our local region and is extrapolating it. You measure the expansion rate of the universe at different eras, you can do that using the relationship between distance and velocity. And that’s where all the problems come from. So all of these problems, dark matter, dark energy, etcetera, are because we don’t make that assumption. They’re not solved by un-making that assumption.

0:17:44.7 SC: Justin Bailey says: Is there a minimum speed of light at which the universe is still interesting, I.e., has beings like us. What would that universe be like?

0:17:52.9 SC: I’m going to un-ask this question a little bit because there’s a different lesson to be learned here. Well, when you say a minimum speed of light, in what units do you mean? Like meters per second? If the speed of light were different, everything else would be different also, it’s essentially meaningless to say if the speed of light were different, the speed of light is always one light year per year, that’s the only natural units you can measure it in, anything else is just invented by human beings. Because if the speed of light were different, the size of the hydrogen atom would be different, all the rulers you use to measure things would be different, so I can always, literally always, choose units in which the speed of light is one.

0:18:38.9 SC: And then, so what you’re really asking is, if I choose units where the speed of light is one, there’s no such thing as the speed of light being big or small, it’s the fact that there is a speed limit that matters, we can always choose units where that speed limit is one, and then you can ask about changing other dimensionless, unit-less constants of nature, the fine-structure constant, the ratio of the mass of the proton to the mass of the electron and things like that, okay. So the right question to ask when you’re asking questions about other ways the universe could have been, is to change dimensionless quantities, quantities that do not have a unit attached to them, like meters or kilograms or meters per second. Any one of those depends on the units.

0:19:24.7 SC: A ratio of, like I said, the mass of the proton to the mass of the electron, that’s the same no matter what units are used, so that could change. The speed of light is a speed, a velocity, meters per second, and there are no other speeds in the fundamental laws of physics, so there’s no ratio you can take to get a dimensionless number. So you just can’t imagine a universe, literally, where the speed of light was anything else, other than infinity, infinity is sort of a limiting case there, right. So the right question to ask is, how can you change other quantities of nature and still have beings like us? And that’s a hard question, that’s one I don’t know the answer to. People have thought about it, but it depends a lot on your assumptions and our calculational abilities, which are not very good.

0:20:10.0 SC: Balarka K says: Is dark matter discussed or framed as a limit to our knowledge in physics?”

0:20:16.3 SC: In some simple way it is, ’cause we don’t know what it is. We know that there is dark matter, we know where it is to some rough approximation, we know how much of it there is, we know how it behaves, we don’t know what actually is making up the dark matter. So yeah, that’s a limit to our knowledge in physics, it’s not framed as a limit that is in principle there, we could easily discover what the dark matter is, what… If it’s a particle or something else, that’s something we’re trying to do very, very hard. So it’s definitely not a limit in principle, it’s just a question we haven’t answered yet, which science is full of those.

0:20:47.6 SC: Johnny says: At a base level are photons much different than any other matter particle? Apologies for the basic question here.”

0:20:57.7 SC: It depends on what you mean. There are certain ways in which photons are different than other kinds of particles and certain ways in which they’re the same. For any way that they’re different, there’s probably other particles that are the same as them, right? So the most obvious difference between particles, photons and electrons, let’s say, is that photons are massless and they move at the speed of light, but gravitons are also massless, they move at the speed of light. Gluons are massless, they would move at the speed of light if they could escape by themselves and just travel through space, but they can’t ’cause they’re confined in strongly interacting particles. So that’s a basic difference, but it’s… Again, gravitons are the same way, so it’s not a unique difference.

0:21:37.6 SC: Photons have zero electric charge, but then again, so do neutrinos, so do gravitons also. Photons have spin-1, but then again, so do W bosons and Z bosons and gluons, right? So you get the point. Photons are different than any other specific kind of particle, but they’re not different than all the other particles combined, in the sense that everything that is different about between a photon and one other kind of particle will be a similarity between a photon and some other particle.

0:22:05.4 SC: Carlos Nunia says: Who’s your favorite superhero and why?

0:22:11.9 SC: I’m pretty agnostic about the superheroes these days, although I’m a fan of the movies, I like the movies, but I’m not a huge comic book fan, I was never, actually, probably a huge comic book fan, I was a medium-sized comic book fan when I was a kid, right. And back then, my favorite heroes probably still today are the sort of most magical ones. So I liked Doctor Strange, right. Doctor Strange, by the way, and for recent Mindscape listeners, one of our early interviews, was with Scott Derrickson, who was the director and co-writer of Doctor Strange, the movie, so we get some insight there into the movie. Thor is another one, as an Asgardian god. On the DC side, Green Lantern, who can do all these crazy things was a member of the Green Lantern Corps and the Guardians of the Galaxy… Not the Guardians of the Galaxy, right. The Guardians of Oa, see, I’m forgetting my youthful knowledge here.

0:23:07.8 SC: But anyway, yeah, I like those sort of way-out superheroes more than the down-to-earth ones, like Batman or Iron Man or Captain America or something like that. Why? Yeah, I don’t know, I don’t know why. I figure if you’re going to be a superhero, let’s do it all the way. Like Superman I, you can fly, okay, good. But I want interesting powers, I want magical powers, basically. That’s what I want. I think that’s what my psychoanalysis here is teaching me. What I really want out of life is magical powers, and I’m sad that the world doesn’t work that way.

0:23:42.8 SC: Sandbox says: Careful viewers of the Veritasium YouTube channel may have noticed a cameo from you in a recent video featuring a bet between Derek Muller and Professor Alex Kusenko, although sadly, we never saw or heard much from you during the event. Can you talk a bit about what your experience being a witness to the contest, were you like Bill Nye persuaded by Professor Kusenko’s presentation or surprised by the outcome of the bet?

0:24:04.4 SC: So some of you might not know this at all, but it’s a kind of a fun story, so Veritasium is a very good YouTube channel run by Derek Muller and who… I’ve appeared on some of the little videos that you can look them up, maybe not the best or most interesting ones, but there I am. And Alex Kusenko is a professor of physics at UCLA, and another friend of mine, and Derek did a video where he traveled in a what looks like a pretty straightforward sail plane… I shouldn’t say plane. What should I call it? Like sail car. It’s on salt flats, and it’s a thing that people have built and has little wheels and a giant sail and a fan, and… Actually, does it even have a sail? Maybe not, just a fan, the fan is sort of acting like the sail.

0:24:47.7 SC: But the point is that the fan is hooked up to the wheels in such a way that… What is interesting is that the sail car, I’m going to call it that, it travels toward the wind, and it can travel so fast, the claim is that it’s actually going faster than the wind is. So if you might know, if you know sailing at all, sailing on the ocean, you can go at an angle to an oncoming wind and you can actually tack overall toward the wind, but the point… The claim here was that this particular sail car, plane car, propeller car, I should call it, was moving exactly toward the wind and was going faster than the wind was pushing it in the other direction. So Alex said, “Nope, that’s not possible. [chuckle] It violates the law of physics.”

0:25:36.3 SC: So they made a bet, the two of them, a $10,000 bet. I hope I’m describing it correctly, ’cause honestly, I didn’t follow it too closely. So the reason why I didn’t follow it too closely was Derek asked me if I would be a witness to the contest, so what that means is they made a bet… And the thing is, you have to trust both of them because there wasn’t any outside panel of jury or jury that would actually judge who won the bet, you were trusting that through the force of reason and through talking about it back and forth, one of them would convince the other one that they were right, that this violated laws of physics or it didn’t. Now, you might say if they did the experiment it couldn’t have violated laws of physics, but you know, experiments are tricky, you can always cheat a little bit, and so Alex in particular thought that what happened was the sail car, the propeller car, prop car, had been accelerated, but then the velocity of the wind temporarily dipped down so that it seemed like the car was going faster than the wind.

0:26:36.8 SC: Anyway, so the point being that they bet each other $10,000 and they said, “Well, we need witnesses to the bet.” So Bill Nye, Neil deGrasse Tyson and I served as witnesses to the bet, so they did a little YouTube video where Alex and Derek pitched their cases to the witnesses, but sadly just because of scheduling conflicts, I couldn’t make it, so that’s why there’s a little cameo there from me, but Bill Nye and Neil deGrasse Tyson were there on the video, but the point is that our job was not to judge who’s right or wrong, it was literally just to witness the bet.

0:27:09.5 SC: In fact, I told Derek, I said, If you want me to judge who’s right and who’s wrong, then I’m not going to do it, because I know perfectly well, this is exactly the kind of physics I’m terrible at, real world stuff, like moving cars and wind and friction and air resistance and all this stuff. I have enormous respect for the people who do that kind of thing professionally, and it’s not what I do, I try to figure out the ultimate laws of physics. If they were making a bet about quantum mechanics, I’d be all over that, but Derek assured me that all I had to do is literally witness, so to keep them honest that they were doing it.

0:27:42.6 SC: And to be honest, since I saw the video, the original video that Alex objected to, I thought probably it was fine. I’m very willing to believe that in complicated real world situations, the intuition of a physicist can go sadly wrong, and eventually, as many of you know, Alex conceded the bet and paid $10,000 to a charity of Derek’s choice, so that’s a lot of money. I wouldn’t bet that much money. But so I honestly didn’t even follow in detail the arguments back and forth, like I said, that’s not what I do, it’s not what I’m good at and not especially interested in following that. I am happy that both of the participants in the bet stuck to the high ground and use the force of reason to figure out what was going on rather than just being, stubbornly sticking to their incorrect position.

0:28:37.1 SC: Lenio Missiora says: In your lectures about the Higgs boson in The Great Courses, you said that because the W and Z particles are originally massless, these particles have to always be moving, and that is how nature knows they are left-handed. However, isn’t movement relative? What about those particles relative to which the Z and W are stationary?

0:28:54.8 SC: So to go back to what we just said about photons a little bit before, when the Z and W were massless, they were moving at the speed of light, and that fact is not relative, that’s the whole point of special relativity, is that to say that something is moving at the speed of light is a non-relative statement, it’s an absolute universal statement. If something’s moving the speed of light, it’s just moving at the speed of light. Literally, you have to be on top of it at the same place to say, “Oh, I don’t see it moving at all.” But then time doesn’t pass for you, so you don’t say anything at all. Okay? So to any other thing in the universe, you’re moving at the speed of light, and that’s why you can point out the direction in which it’s moving, it is always moving along in some direction, and then in that direction, it’s either spinning left-handed or right-handed.

0:29:44.1 SC: Mikolas Szabo says: Is it true that current quantum computers are not the ENIACs or EDVACs of quantum computation, but more like Babbage’s analytical engine, or is even this a stretch? What are the chances that quantum computing won’t work in the end as Wolfram predicted in your interview with him?

0:30:00.6 SC: Well, I don’t know if the analogies are just very good, in this case, it’s not that it is ENIAC or Babbage’s analytical engine, it’s just a different thing, because back in those days, number one, we didn’t really have a theory of computation that was very good, and number two, we didn’t have other computers that worked, they were trying to build the first computers. Building the first quantum computers is putting together two great tastes that taste great together, but are individually perfectly good: Quantum mechanics and computers. Computers work, quantum mechanics works. I think that there’s essentially zero chance that quantum computing won’t work in some deep principled physics reason.

0:30:40.7 SC: I think it’s perfectly possible that quantum computing will be not as exciting as we hoped because of some practical technological reason, if it just becomes too difficult to connect enough qubits with a short enough interaction time between them and enough reliability, etcetera, then it will be possible to build a quantum computer, but so prohibitively expensive that no one will do it. I don’t think that’s likely, but I think it’s conceivable. I don’t think… I don’t remember exactly what Stephen Wolfram was saying, but I don’t think there’s anything lurking in the fundamental laws of physics that prevents quantum computers in principle.

0:31:14.2 SC: Robin Van Dyke says: My question is about how you come up with ideas for writing a paper. Is it just by talking with other physicists about your work or are there other things that influence what you write papers about?

0:31:27.8 SC: Yeah, there’s lots of things. There’s no… It’s exactly like asking a novelist where they come up with ideas, there’s no algorithm, there’s no recipe, you don’t just plug in say, “Well, I would like an idea now.” Of course, you talk to other physicists, you read other people’s papers, you go to their talks, you go to conferences, you talk to your students and your colleagues and your postdocs, you read books, you read things elsewhere, and you just think, right, just sometimes you just sit and think about the puzzles that you have been bothered by for a long time. And the problem is never coming across a puzzle, the problem is coming up with an idea that is sufficiently promising to answer a puzzle or to address a question that you can push it forward in some practical way.

0:32:13.1 SC: It’s easy to come up with questions, it’s easy to propose answers, it’s hard to come up with answers that are… We don’t know enough about it that it’s worth doing work on, but nevertheless, we can imagine making progress on it, that’s the crucial sweet spot in there, you want a problem that is not necessarily solvable, but at least progress can be made in some sense.

0:32:35.9 SC: Okay, and the next two questions, I’m going to group together. They’re about gravitons. Stu Haynes says: I believe you’ve said that we can be a bit loose and describe gravity as a force, even though it is really the curvature of spacetime. If so, why do we speak of graviton as a particle mediating the force of gravity? Peter Sulfez says: Are gravitas merely the particle duality of gravitational waves? If not, why is there so much confidence that they exist?

0:33:01.1 SC: Actually, I’m going to stick another… One more question in here, in this grouping. Andrew Juul says: There are four forces, with gravity and electromagnetism, there’s a simple formula to calculate the force and plug in to F = ma, but with the weak and strong forces, they’re simply strange phrases, such as is involved in radioactive decay and never an equation. So what does force mean with the strong and weak forces, is there a way to calculate an F, which can equal ma? So all very good questions in the sense that I have answers to them, that’s always a favorite situation to be in.

0:33:31.8 SC: So why do we speak of gravitons as the a particle mediating the force of gravity? Well, you can speak about the same thing in two different languages, right? So that was for Stu’s question. For Peter’s question, are gravitons merely the particle duality of gravitational waves? Yes, that’s what they are. So there is the limit, if you like, of physical reality, where waves in either gravity or in electromagnetism are big classical things, and in that limit, it is perfectly sensible to treat these things as classical fields, the gravitational field, the electric field, the magnetic field, ripples in these different fields are what we’re talking about, and you’re in the macroscopic world, and you can use F = ma and you can have an equation and figure out the force moving around a particle.

0:34:17.6 SC: The particle-like nature of these forces comes about when you get down to the quantum mechanical level, so when you’re just talking about a little tiny bit of the force, when you’re measuring just one little packet of energy. For photons, that’s easy to do, we can see spots on the CRT screen or something like that. For gravitons, the force is too weak, so technologically, we can’t do that. But the point is, there is a regime in which it’s perfectly sensible to think of these guys as particles; there’s another regime in which it’s perfectly sensible to think of them as big classical fields, giving rise to big classical forces. At least, that’s the case for both gravity and electromagnetism, because these are long-range forces. For the strong and weak forces, to Andrew’s question, there is no long-range limit, they can’t pile up because they’re short-range forces, they just don’t stretch over that large of a distance.

0:35:07.4 SC: So you never reach this limit where you have a big macroscopic strong nuclear force field or a weak nuclear force field, it just never happens in the physical world, but at that quantum level, there’s a huge amount of similarity between the strong force, the weak force, the electromagnetic force, the gravitational force. There are equations, and the equations are very, very much alike for all of those four different possibilities, but it’s just the big classical limit doesn’t exist because the forces are such… Are so short-range is what I’m trying to say. So, should you call them forces then, the strong and weak forces? Eh. I don’t care. I honestly just don’t care, but they’re very analogous to electromagnetism and gravity, so in that sense, I see why someone would call them forces.

0:35:55.2 SC: But you’re right, we don’t actually just take a particle and plug it into an equation and say, “Here is the force,” but you could, for either the strong force or the weak force. The problem is that equation would just never apply to any big macroscopic thing, that’s why you don’t actually see it in everyday life.

0:36:14.2 SC: Felix Roswald says: Is there a one-to-one ratio of electrons to protons in the universe? Or in other words, is there a net charge to the universe?

0:36:22.5 SC: So I’m going to undo the second half of your question a little bit because… Well, I’m going to separate these questions, it’s not in other words, because of course, there are other particles that have electric charge, muons have electric charge, so muons can decay into electrons, neutrinos and anti-neutrinos, or vice versa, I can smash an electron and a positron together and get a muon and anti-muon. So I can change the number of electrons without changing the number of protons, so there’s not exactly a one-to-one ratio of electrons to protons, but if what you’re really asking is, is there a net charge to the universe? Well, “we don’t know” is the short answer. There’s very few questions about the universe as a whole to which we know definitive answers, but most of us think there is no net charge to the universe.

0:37:11.5 SC: There’s a theorem that says that if space is closed, so if the spatial slices of the universe are spheres or a torus or something like that, then there can’t be a net charge, then there has to be zero net charge, that’s a consequence of Gauss’s law or something like that, but if the universe is open, yeah, then you could imagine a universe with a net charge. There it becomes trickier and we just don’t know.

0:37:34.8 SC: Anonymous says: I really enjoyed your discussion in the last AMA about the difference between being a public intellectual and being an activist. I thought about it some more, and I realized that the two roles aren’t completely disconnected. An intellectual whose conclusions just aren’t palatable to the average person’s sense of decency won’t be seen as very credible. An activist whose conclusions just aren’t realistic won’t be seen as credible either. Suppose that an intellectual needs to think like an activist sometimes, where would you set the boundary between staying in touch with the society we live in on a basic level and letting personal feelings and wishes get in the way of objectivity?

0:38:10.0 SC: So this is a very good point, and I think that I was probably sloppy when I was giving the discussion, even though I, for purposes of explanation, distinguished between a kind of person called an intellectual and a kind of person called an activist, it’s much more accurate, as you’re getting at, to say that there are roles in which any one person sometimes plays the role of an intellectual trying to figure out the truth, and sometimes plays the role of an activist, and there’s probably zero people who are 100% one or the other, there’s always a little bit of both. So “how do you balance that” is basically what you’re saying. Where would you set the boundary in between living.

0:38:52.2 SC: And different people are going to do it differently. I think that… I am personally most comfortable being much more of an intellectual than an activist, but I do think that it’s important to make the world a better place, if we can. I just don’t feel comfortable with doing that in ways that would be anti-intellectual, that would involve… What I said was, if your goal is primarily to change the world, then I can absolutely see circumstances in which doing something other than telling the truth would be involved. But if your goal is a little bit to change the world and a lot to tell the truth, then I don’t see why you should ever not tell the truth. Do your best to change the world in ways that are compatible with telling the truth. That’s what I would try to do.

0:39:41.3 SC: Kyle Morris says: Why isn’t gravity or the gravitational field constantly decohering quantum systems and getting rid of their wave-like natures? Seems like even a single particle would be constantly updating spacetime with its position and energy.

0:39:54.4 SC: Well, this is a subtle question, actually, and I don’t think that the answer is completely understood, but let me just add a little bit of a true fact that might help you see what’s going on. Think about the sun. That’s a big quantum particle. It’s a very big quantum particle, in the approximation where you’re ignoring the internal degrees of freedom of the sun, and it has a gravitational field. But the gravitational field sort of exists as part of it. The gravitational field isn’t changing. There’s just the sun and the gravitational field of the sun. It’s like kind of one system, right? So the sun isn’t… In the approximation, where the sun is just sitting there alone in the universe, there’s no dynamics.

0:40:33.1 SC: It has a gravitational field, full stop. That’s all that’s going on. Decoherence is of necessity a dynamical process. Decoherence is the transition from two systems being un-entangled to two systems becoming entangled, where one of those systems is the environment, right? So the gravitational field of a single object isn’t the environment. An environment, you should think of as a lot of propagating degrees of freedom, like a lot of photons or a lot of atoms or something like that. It’s not the field that is attached to some object. So it’s not the gravitational field, per se, that would be doing the decohering, but you could imagine gravitons doing the decohering.

0:41:12.4 SC: Like if you moved the sun and it gave off some gravitons, those could sort of scatter into the environment, and if they then interacted with other things in the universe, then you could decohere. But you need a lot of that, right? Gravity is very, very weak. So it becomes a quantitative question, you actually have to calculate how quickly it happens, and that turns out to be hard to do, which is why, like I said, it turns out to be a subtle question.

0:41:37.0 SC: Okay, Lester Sue says: “I think you’ve said before that all good Bayesians should update their priors in the same way based on objective new facts. But if a Bayesian starts out with their subjective priors, surely those priors would and should also inform how they assess new facts, and therefore, how that will update those priors. Or is there an objective standard to how you assess new facts as a Bayesian independent of your priors?

0:42:02.2 SC: So I’m not quite sure that I’m interpreting the question correctly, but I’m going to interpret it in a way that I think is a charitable way, in the sense that I’m going to interpret it in a way that I agree is a really good issue to worry about, namely, none of us is perfect Bayesians. In fact, even the thought experiment of Bayesian reasoning is just something that does not map onto how the real world works, okay. Because what Bayesian reasoning says is, you have your priors to various different propositions and those are, in some sense, subjective, okay. Different people are allowed to start with different priors. They think that some things are more likely, some things are less likely. But you’re not allowed to pick your own likelihood.

0:42:41.5 SC: So the likelihood is the theoretical prediction, given the different proposition you want to believe in, what is the probability of getting certain data coming in, certain new information. And we pretend that those livelihoods are completely objective, but in practice, they’re not, because often the propositions we’re trying to judge in some way are not perfectly well-defined, okay. So previously, we were talking about the likelihood of life existing in the universe under theism or atheism. So I’m happy to say the probability of life existing under theism is very, very large.

0:43:17.5 SC: It will be… What would be the point of having God around be if God didn’t create life somewhere? But I don’t know what the number is. [chuckle] I don’t think that the phrase theism attaches to any sufficiently well-defined theory to let me predict what this purportedly objective likelihood function is, right? So that’s one real-world shortcoming of the Bayesian paradigm. And the other is what I think you’re getting at, Lester, which is that… Well, there’s two things, but both of them have to do with the way that we take in new data or new information.

0:43:54.3 SC: One is when we get some new information, what is the credibility that we give to that information? Maybe our brains are misleading us, or maybe we’re just being lied to or whatever. And the same amount of information might be judged to have different amounts of credibility by two different people. So in principle, that is something to worry about, but it’s something Bayesian reasoning is perfectly adapted to taking care of. You sort of take that into account when you define what is your proposition, what is your likelihood, what is the data, and all that stuff, right? All of these have different probabilities associated with them and they all get massaged in to the final answer in the correct way.

0:44:33.8 SC: An even more interesting issue to me is that we don’t take in the same information, right? And this, again, goes back to the question that we were talking about before with theism versus atheism, because if the only thing you looked at was the fact that there’s life in the universe, you might get one answer, but then if you also look at the fact that there are many other galaxies and many other stars, the universe is big, and it’s old, and it’s a mess, etcetera, etcetera, then you might get a very different picture. So increasingly, and this isn’t directly related to your question, but it’s just something I’ve been thinking about myself, I’m impressed… Impressed might not be the word. I care a lot about the fact that we are computationally bounded, as Stephen Wolfram said it, although I was thinking about it for other reasons with other words attached to it, but we’re finite, right?

0:45:20.9 SC: We don’t take in all of the data. In Zeynep Tufekci’s words from several podcasts ago, attention is the precious quantity that we are talking about now, not information. There’s too much information out there. What do you pay attention to? And since we are tiny finite people, we need to choose what to pay attention to. And so even two people who agree on all of the priors and agree on all the likelihoods might choose to pay attention to different inputs, and therefore get different data that they use to update their Bayesian credences.

0:45:58.8 SC: And so, this is not to say that Bayesian reasoning is wrong in any way. It’s not wrong. It’s an idealization of what really happens in the real world. And so I think that there’s probably… And maybe it’s already been done. I don’t know. I’m not an expert in these things. But there’s probably a lot of work to be done or that has been done on bounded rationality, right? I think that’s a technical term for it, the fact that we are not perfect either in reasoning or information-gathering, and therefore, what’s the best you can do under those circumstances? There’s probably been a lot written on that that I don’t know about. So maybe we should have someone on the podcast to talk about it.

0:46:35.8 SC: Fabian Rosedallan says: Regarding your work in very fundamental questions of the universe, have you ever had to deal with any existential anxiety or pain or anything like that? Do you have any thoughts about how quantum physics could influence these matters either positively or negatively?

0:46:50.9 SC: So no, and no, are the short answers here. I think I’ve said this before. When I was a kid, when I first started thinking about cosmology and things like that, there was a little bit of anxiety involved, and it was specifically with the question of, “What if I hadn’t existed?” [chuckle] Unbeknownst to my young self, I was thinking in… I was doing modal reasoning. I was thinking about the space of all possible worlds, and there are possible worlds in which I didn’t exist, or there are possible worlds in which the universe doesn’t exist. Should we call that a world? I’m not sure. But anyway, that always seemed to be like the end point of where I could sensibly reason about things, and that made me nervous or maybe even anxious.

0:47:35.4 SC: But then, yeah, so I think that now, when I’m thinking about those questions, I’m just not sure that… I talked before, I did a whole podcast solo episode about why is there something rather than nothing, and the answer that I advocated there is, that’s not a question to which we have any right to think that there’s an answer. We can formulate questions in what appears to be a grammatically legitimate way, “Why is X true?” and very often we can get answers to those questions, but it turns out that those answers are embedded in a context, right?

0:48:08.0 SC: “Why is my car out of gas? Well, because I was going to put gas in it yesterday, but I forgot.” Or “Why is my car out of battery since I have an electric car?” But these are all in that context. And the question, “Why does the universe exist?” doesn’t have a kind of context in which there would be a sensible answer. So it’s not the answer is we don’t know, or the question is outside our powers to address or anything like that. It’s just that… It’s not a good question. It’s not an answerable question. It’s not a question that has an answer. So that tends to be more my attitude these days with the kinds of questions that would lead to existential anxiety.

0:48:47.6 SC: And quantum mechanics has nothing to do with it, as far as I can tell. I’m not really sure why it would. Especially for someone like myself who is a thorough-going physicalist about quantum mechanics. There’s a quantum state and it obeys an equation. I’m not sure why that would change my existential anxiety over classical mechanics in any way.

0:49:06.7 SC: Ratboy.exec, probably also not their real name, says: I recently listened to your talk with Philip Goff on panpsychism and was absolutely enthralled. One thing that struck me was the discussion on intrinsic properties, those that don’t govern observable behavior and aren’t captured by mathematics. It reminded me of your discussion with Max Tegmark who seem to argue there’s no such thing, to the extent that the universe is nothing more than an abstract mathematical structure, and even further that this universe is not uniquely real among mathematical structures. My question is this: If your position is that there are no intrinsic properties, what in your view distinguishes the world from the abstract mathematical structure described by Tegmark? And if, in contrast to Tegmark’s view, this universe possesses a unique property called existence, would that qualify as a sort of intrinsic property that we get to be privy to by virtue of our being internal to the structure?

0:49:55.3 SC: So I think that it’s hard to give a completely satisfactory answer to this without a rigorous definition of the phrase “intrinsic property,” okay. It’s one of those things where we think we know what the word “intrinsic” means, we think we know what a property is, and therefore we think we know what the phrase means, but it gets a little bit slipperier upon further examination. So in mathematics, in set theory, there is a definition of property, and it’s a little deflationary, it’s just a subset, [chuckle] right? You have a set of many, many things, and we’ll talk about a property that some things have and some don’t. You just enumerate the ones that have the property.

0:50:38.2 SC: And so the property is simply equivalent, isomorphic to the subset. That’s what a property is in set theory. So I could take that attitude about properties of physical things too. I could say properties are just a way of saying like, if I say the property of moving at the speed of light is just the set of all things in the universe moving at the speed of light, right? And then it gets a little bit tricky to do that carefully, but something like that could happen. So you’re adding the adjective “intrinsic” to property. I don’t know what that means. I don’t know what that is supposed to mean, especially if you’re saying that it’s an intrinsic property that doesn’t change its observable behavior in any way.

0:51:21.4 SC: I know what properties are. I know what electric charge is. I know what mass is. And the reason why I know what they are is because they play a role, dynamically. They appear in the equations of motion that predict what’s going to happen to this physical system. If you say, “Well, I have another kind of property that’s really important, but it has no effect whatsoever on what happens,” [chuckle] I’m not really moved by that, right? Now, so you’re trying to pick out the specific example of existence as a property, and I’m not sure what that means exactly, so I am… And this might be my failing, is I’m not trying to make fun of you, I’m making fun of myself here. I’m opening up to things that I haven’t thought about very deeply, so I could be completely wrong, but I’m going to give you my initial impression.

0:52:08.1 SC: I do think that there is something called the universe. My phrase from my own point of view about this is reality realism. What do I think is real? Reality. What is that? Well, I don’t know, but we’re trying to figure that out. That’s what science is trying to do. What we try to do is model reality, and when we model reality, we do so usually with a mathematical formal system of some sort. And so I am not a Tegmarkian in in the sense that I think all different mathematical systems exist for reasons, that I tried to make clear during that podcast. I think that the real world exists and it’s described by some mathematics and not others, okay.

0:52:42.5 SC: So, could you think about that as saying, “Well, consider the space of all possible worlds and point to one and say, ‘This is special ’cause it has the property called existence'”? Sure, I think so. I think that’s a possible thing to do, but I don’t think it’s a necessary thing to do. I could just say the real world exists, and I could stop. I don’t need to compare it to all the other non-existent worlds. So it’s very useful. That makes me not a modal realist. Modal realism, long before Max Tegmark, was this idea of David Lewis’s that all the alternative possible worlds exist in some real sense, and all we try to do is figure out which one we’re in, okay. I think that’s a very good reasoning strategy.

0:53:26.3 SC: I think it’s very… When we’re asking ourselves, “What are the correct laws of of physics?” that is exactly equivalent to saying, “Consider all the possible worlds with different laws of physics, which one are we in?” But I don’t think that anything is added by saying that the other worlds really exist, or… So I think that our world exists and that is different, but it’s not a property that our world has, it’s just the world, there’s nothing extra to it. Now, if you don’t understand the many worlds of quantum mechanics very well, you might think that this is hypocritical, because I do think that the other worlds exist in quantum mechanics. But that’s because they are causally connected to our world. They were part of our world before and they split off, and there’s an equation that describes that. It’s not simply a posit that says, “Imagine they’re all there.” They’re a dynamical prediction of the Schrödinger equation. I hope that was an answer to your question. I’m not sure, but I did my best. [chuckle] Okay.

0:54:17.9 SC: Ken Wolf says: Back in the day, we were all taught that there were just three types of chemical bonds, covalent, ionic and metallic. I recently read an article by Philip Ball about how new research suggests there might be new types of chemical bonds such as ones that blur the line between covalent and metallic. This got me to wondering where any new developments in quantum electrodynamics could possibly give us new insight into how chemical bonds might work? Do you think this is possible? A shorter version of the question could change us in the standard model, change our understanding of how chemical bonds work, or is that pretty much locked in by now?

0:54:52.1 SC: I think that’s pretty much locked in by now. This is part and parcel of the idea that the laws of physics underlying our everyday lives are completely understood, right? The core theory is what we have. And roughly speaking, it’s completely possible that we will learn more about chemical bonds than we know now. But the laws of physics that give rise to chemical bonds, the fundamental equations of quantum electrodynamics and the standard model, those are there, they’re not changing.

0:55:19.9 SC: And the reason why we know is because we can characterize what kinds of possible changes there are, and we can look for them, we can do the experiments to see whether they’re there, and so far they’re not, and we have pushed any possible modification of the standard model out to regimes where it very well could be there, but it would have no effect whatsoever on chemistry or anything like that. Think about the podcast I recently did on the muons, on different possible, we don’t know, but possible new physics signatures of how muons decay and interact in their magnetic moments. These are so incredibly tiny, we’re not even sure they’re there, but they have zero effect on chemistry or anything like that. And that’s the only thing that we have seen that could possibly be… Or let’s say. It’s the most reasonable, most likely deviation from the standard model we’ve yet seen. So that is my opinion about that. I could be wrong.

0:56:20.3 SC: The thing about saying things like this is one could always be wrong, but one has to do the best one can do about assigning credences to different possibilities. My credence that new fundamental physics is going to change chemistry is very, very, very tiny indeed.

0:56:36.5 SC: Ryan Morrison says: I’m not an expert in psychology, but I think Maslow’s Hierarchy of Needs and the idea that we can make progress as humans is pretty attractive. Do you ever think about the idea of transcending the self, or have you moved away from that kind of thinking? Do you think it’s possible to be totally fulfilled?

0:56:52.2 SC: Well, luckily, Ryan, I did a whole podcast on that with Scott Barry Kaufman, and Scott is a psychologist who actually is a huge fan of Maslow, and he has updated Maslow’s Hierarchy of Needs in a way that I think is really much better. I get what you mean. The Hierarchy Needs is an attractive notion, but there’s something that is missing about it. And so what Scott points out is, number one, Maslow never drew the pyramid. [chuckle] You know, the famous pyramid picture, that was done by later commentators. And the pyramid as a metaphor is a little bit misleading ’cause it’s a little too solid, right? It makes you think there really is a structure that we’re climbing to get to transcendence at the top of it, whereas the world, the reality in our lives is much more dynamic.

0:57:38.6 SC: And so Scott is trying… Is arguing for replacing the pyramid metaphor with a sail boat metaphor, which I like. Basically, I’m totally a fan of this way of thinking. So no, I don’t think it’s possible to be totally fulfilled. I think that’s a misunderstanding of what it means to be alive and human. To be alive and human is to be a process, to be changing. Changes might be under the surface, or they might be very obvious, but it’s completely wrong, in my view, to imagine that there is one state that we just need to get into, and then our work is done. I think that’s the opposite of how we should think about living our lives.

0:58:15.6 SC: Casey Mahone says: I’ve been thinking a lot about your notion of emergent spacetime. In my mind, it raises a similar issue as the hard problem of consciousness. In both cases, it seems completely unclear to me how something of a totally different kind can arise simply through emergence.

0:58:29.5 SC: Well, I’m sorry that it seems unclear, but it happens all the time. I mean, that is one of the great things about emergence. One of the things that you learn about emergence is, the rules of the emergent theory can have no obvious connection to the rules of the underlying theory. There will be a non-obvious connection, there might be some subtle connection, and it will be there because they have to be ultimately compatible, but as we say, the ontology, the different kinds of stuff you have in the emergent theory can be totally different.

0:59:00.4 SC: The most obvious and well-thought about example is quantum mechanics and classical mechanics. In quantum mechanics, this stuff is a wave function of vector in a Hilbert space, that’s when you start with, and what you see at the end are particles of classical mechanics, particles and fields living in spacetime, a completely different kind of thing. And forget about emergent spacetime, okay, just quantum mechanics, just good old quantum mechanics, the hydrogen atom, right?

0:59:29.2 SC: The hydrogen atom, the sort of picture that we draw, the Rutherford atom, the little dots in the middle that represent the nucleus and little dots orbiting to represent the electrons, that gets us a certain place, but it’s not the underlying quantum reality. Once you get to the solar system, same thing, the solar system is very well-represented as a big sun with rocks, planets orbiting around it, but the real thing is an underlying quantum wave function. Emergence just does that all the time. The question is, is there a map from the underlying microscopic states to the emergent macroscopic states? And the map can relate to very, very different theories.

1:00:08.0 SC: Rob Griber says: This month I wanted to ask a question for my seven-year-old daughter, Liana.” So this is a hint to other future AMA askers, I’m a sucker for these kinds of questions, so please don’t lie, but if you really want a question answered, it does increase your chances if the question comes from your seven-year-old daughter. The question is: Professor Carroll, if you were standing on the photon sphere of a black hole and you looked forward, you would see the back of your head. Is that right? What would happen if you looked up? What would you see? And what would you see around you to the left and the right?

1:00:43.6 SC: So I want to be a little bit careful here, because there’s the real world and their sort of physics idealized play land, okay. So there is, in the world of black holes, something called the photon sphere. This is the region around the black hole, a little bit outside the event horizon, where a photon could travel exactly, roughly speaking, exactly parallel to the event horizon at a constant distance above it, okay. So a photon could just go in a circle and you call it the photon sphere. It’s outside the event horizon, so it’s not the region from which you can’t escape. If you are on the photon sphere and fire your rocket engine, you could escape, okay, but if you are just there and you are not a photon, you could send out a photon in exactly the right direction, and it would go around and you would indeed see the back of your head.

1:01:32.8 SC: That’s the story we tell, anyway. In the real world, there’s an extra fact that becomes very important, which is that these orbits of photons on the photon sphere are not stable. If you deviate the direction in which the photon is pointing by any tiny bit at all, the photon will either fall into the black hole or zoom off to infinity. So in the real world, there’s going to be almost no photons on the photon sphere. Almost all of them that might have come close will either fall into the black hole or zoom off to infinity. So you probably won’t see anything special at the photon sphere in the real world.

1:02:09.8 SC: But let’s go along with a thought experiment. What happened if you looked up, or left or right? Well, I’m not exactly sure what direction you’re pointing in here. Of course, in outer space, there’s no such thing as up, down, left or right. All the directions are created equal. So basically, if you’re next to the black hole, there’s only two options. You’re either looking… Well, I guess there’s three options. You’re looking toward the black hole, away from the black hole, or perpendicular to the black hole, right? And if you’re looking along the photon sphere, you’re looking perpendicular to the black hole. So it doesn’t matter whether you look straight or up or backward, in any of those directions, you’re looking perpendicular to the black hole and you would see the back of your head.

1:02:52.1 SC: If you look toward the black hole, you see nothing. It’s black, roughly speaking, you could see something, it was in the process of just falling into it, that’s very possible, but the black hole itself is just black. If you look the other way, you see the real world, the outside world. You see all the stars and your friends back in the spaceship who are thinking to themselves, “What are they doing messing around near the photon sphere? They’re going to be in trouble.” But otherwise, it’s perfectly normal. So in fact, to a large extent, it is just perfectly normal. Don’t think of the photon sphere as a very dramatic place that you would notice if you were visiting the vicinity of a black hole.

1:03:26.9 SC: Jesse Rimler says: Music emerges from the naturally occurring overtone series, which is the same everywhere in the universe. The scales and chords we build out of this vary from culture to culture. As a thought experiment, I tried to imagine a fundamentally different kind of musical harmony beyond cultural or temporal differences. Am I right to conclude that it would have to be a universe with a different kind of math, since the octave and the overtone series are based on mathematical relationships?

1:03:53.0 SC: Well, I think the real answer that I have to give here is I don’t even know what it would mean to talk about a universe with a different kind of math. I can imagine talking about a universe with the different kind of physics, different laws of physics, but math is math. Math goes back and forth between universes. Pythagoras’ theorem follows from Euclidean geometry, no matter what universe you’re in, and likewise, the fact that 440 divided by 2 is 220 is going to be true in every universe, right? So I don’t think it’s a matter of different kinds of math. There could be different kinds of laws of physics, but then there might not even be sound or something like that. So I’m not sure what we’re talking about.

1:04:33.7 SC: So I don’t think looking at different kinds of math is the right way to go here. I think that looking at different relationships within the known laws of math, between different frequencies and different combinations of frequencies might be the way to go. I mean, an obvious way to go is to go beyond the idea of discrete individual frequencies, right? It’s usually just to keep our lives simple, we build instruments that mostly play one note at a time, it’s not always true, of course, you can think of counterexamples to that, but typically, a note on an instrument is a note that is centered in its frequency, in its spectral resolution, around a single frequency, and going beyond that to more complicated spectra of sounds coming out of a system might be an interesting way to go. I don’t know. I’m just making things up.

1:05:22.7 SC: Michael Adelman says: In the last AMA, you dismissed the idea of nations having intelligence or consciousness, likening it to anthropomorphizing. Given that consciousness appears to be an emergent property of sufficiently large networks of neurons, why shouldn’t we also see conscious states arise from networks of more complex organisms like humans and animals? George Dyson suggested this in his Darwin Among the Machines 24 years ago, drawing parallels to Hobbes’s Leviathan.

1:05:48.7 SC: So I forget what I said last month in the last AMA, but in principle, sure, I mean, in principle, you could have giant consciousnesses made out of tinier pieces that played the role of neurons or something like that. But the fact that it is possible in principle doesn’t mean it actually happens in practice in the current world. I think that the consciousness that we have in our human brains is pretty dependent on special properties of our particular human brains. There’s other ways, I’m very sure, that you could have consciousness, but it’s not just going to pop into existence without any effort, right? We are the result of billions of years of evolution. Most of those billions of years, we were single-cell organisms, but still, there were selection pressures that were shaping what we have in our brains and in our nervous systems and in our biology to compete in certain environments and to reproduce and so forth, and the ultimate result of that is the capacity that we have in our brains to do information processing in a very coherent way, in a way that gives us a sense of self, a sense of I.

1:06:57.6 SC: Whereas a nation is a bunch of people that is much more loosely organized than that, with no special structure, no special rules of engagement and ways that different connections are drawn in different sub-structures like the brain has and so forth. So it would be astonishing to me if real societies were sufficiently similar to human brains that we would call them conscious organisms. And indeed, I don’t think that’s how it is. When you actually want to convince a nation to do something, it’s much more useful to convince the boss of that nation to do something then try to convince the nation as a whole. One of the aspects of why nations are very different than brains is that brains don’t have a president or a king or an emperor. None of their neurons is the king neuron. That just not how it works. So yeah, in principle, the laws of physics do not prevent nations from being conscious, but in practice, I think it’s kind of a different thing.

1:07:53.7 SC: Peter B says: One of your earlier guests, Leonard Susskind, has argued that inflation must create infinite open universes with negative curvature, yet you have mentioned that we live in a de Sitter space with positive curvature, so I’m confused. Are we living on a giant Pringle chip or a giant balloon?

1:08:10.8 SC: Well, so I don’t know, this is possibly my fault, but it depends… You have to distinguish between the curvature of space and the curvature of spacetime, okay. That’s one thing. The other thing is, what Lenny is saying about the negative curvature, we don’t know if that’s true or not, okay. So the difference been spatial curvature and spacetime curvature is… And we have cosmology. In cosmology, there is a special situation, which is not necessarily true in other aspects, in other situations, let’s say, in general relativity, where spatial slices are homogeneous and isotropic, okay. There’s a special way of slicing spacetime into space plus time, so that the spatial sections are more or less the same everywhere on large scales.

1:08:57.4 SC: So unlike just an empty flat spacetime, Minkowski Space, in cosmology, there is a best way of slicing spacetime into space plus time. So then given that, we can talk about the curvature of space, as well as the curvature of spacetime. The curvature of space is pretty close to zero. We’ve measured it. We’ve not measured any overall deviation from zero. So it could be very, very small but positive, or it could be very, very small but negative, or it could be zero to any practical precision.

1:09:33.4 SC: So, what Lenny is pointing out is that in his favorite version of eternal inflation, where there is bubble nucleation, where a little bubble of true vacuum, or at least lower energy vacuum comes into existence inside a high energy, false vacuum state, you will very naturally get spatial slices that are negative curvature. Now, the spatial slices can be negative curvature, whether or not spacetime is positively or negatively curved. And as we said before, there’s different kinds of curvature that spacetime can have. There’s Ricci curvature and Weyl curvature, etcetera. So when you even say positive and negative, you’re already simplifying things an enormous amount. But the point is that our universe is headed toward being empty other than for a positive cosmological constant, right?

1:10:26.7 SC: That’s the future of our universe. There’ll be nothing left but the positive cosmological constant, and that will be a de Sitter geometry overall, a spacetime with a positive curvature. That’s completely compatible with the surfaces of constant energy in cosmology being negative curvature. There’s a difference between the curvature of the overall spacetime and how you slice it into space plus time. So the two are actually compatible. It’s also possible that there are other ways of doing eternal inflation, like what Jennifer Chen and I looked at years ago, where you have baby universes being created that nucleate as spherical bubbles off of the parent spacetime, in which case, the spatial sections would also be positive curvature. So we don’t know whether space is positively curved, negatively curved or zero. We do have good empirical evidence that spacetime is headed toward a positive curvature state.

1:11:25.4 SC: So Mark Zug says: Regarding photons interfering with themselves, what is wrong with the idea that electromagnetic energy must travel as waves but only be detected as particles?

1:11:33.2 SC: Nothing. Nothing wrong with that idea. That’s perfectly sensible. It’s an incomplete idea, because it’s kind of mixing classical language with quantum mechanical language. Classically, the electromagnetic field is a wave, right? It’s an electromagnetic wave. And then if you add a little bit of quantum mechanics in, you say that when you detect the electromagnetic wave, you detect discrete packets of energy, AKA particles. Now, if you’re a little bit more careful, if you’re going fully quantum mechanical, then that original electromagnetic wave was itself quantum mechanical. It was the quantum field wave, okay. And that’s a little bit more complicated, but still it was a wave. So yeah, I think that as a rule of thumb, saying that EM waves travel as waves but are detected as particles is more or less on the right track.

1:12:24.8 SC: Mark Matthews says: Could quarks in a proton run into their virtual anti-quark counterpart and exchange places with a virtual quark mediated by a gluon resulting in a virtual quark to replace the original quark?

1:12:37.9 SC: So okay, there’s a bunch of things going on there. I’m not quite sure what the picture is, but the reason why I wanted to answer this question was just to talk a little bit about this thing that we always say, that quarks, and neutrons for that matter, that protons and neutrons for that matter, have three quarks in them. In the case of a proton, two ups and a down, in the case of a neutron, two downs and an up. It’s not really like that, okay. Anyone who’s taken quantum field theory or thinks very carefully about particle physics knows better. This is a very sloppy way of talking about protons and neutrons. The reality is that protons and neutrons inside are a bubbling sea of… Actually, that’s not even a reality. A slightly better metaphor is that inside protons and neutrons, there’s a bubbling sea of quarks and anti-quarks and gluons, okay. There are virtual particles, and it’s not just like the occasional virtual particles, it’s mostly virtual particles.

1:13:33.0 SC: There’s a whole bunch of things going on. Now, the reason why I say that’s not exactly right is that even that is speaking too classically and using a particle-like language. What’s really going on are there are quantum fields that are not separable into individual particles. That’s the truth of it. Usually, you don’t want the truth. Usually you want the picture. And so, the vivid picture is a roiling sea of particles popping into and out of existence. The reason why that’s not a great picture is that it gives you the idea that inside the proton and neutron, things are changing over time, things are popping out of existence, and they’re not. The inside of a proton, neutron is static. Nothing is happening inside there. If you were to observe it, you would see different things at different times, but that’s because of the disturbance that you’re putting on it by observing it. Anyway, if you want to speak this particle language at all, it is much more accurate to think of it as a whole bunch of quarks and anti-quarks and gluons always interacting and popping out of existence and creating and annihilating and so forth.

1:14:38.5 SC: And what practice particle physicists will do is talk about the valence quarks inside a proton or neutron. What they mean by that is, when you typically say there are two ups and a down in a proton, what you mean is if you count up all of the quarks and all of the anti-quarks and all of the gluons, on average… Sorry, not on average. There’ll be lots of quarks and anti-quarks, but there will always be precisely more, three more quarks than anti-quarks inside a proton, okay. That’s the point. And the reason why you know it’s precisely three is because there’s a Baryon number that is 1/3 per quark and the total for the proton is equal to one, and that’s true, okay. There is 1/3 Baryon number per quark and the total has to equal three… Sorry, the total has to equal one, my brain’s not really working today, man. I’m going to get to harder math in a second, so I’d better increase my mathematical skills here.

1:15:37.4 SC: 1/3 x 3 = 1. There we go. So there’s an excess of three quarks over anti-quarks in a proton or a neutron, and there’s in fact an excess of two up quarks and one down quark over their anti-particles. But there’s a tremendous number of quarks and anti-quarks popping in and out of existence all the time, if you want to use that virtual particle metaphor at all. That’s the best I can do without going into the equations.

1:16:04.3 SC: Michael Ailing says: Can you suggest one or two books that give an overview of the philosophy of science and highlight some of the current issues those in the field are tackling?

1:16:13.3 SC: So sadly, my answer here is no, [chuckle] I cannot suggest those books, but let me explain why. There are two very different kinds of things, related, but different kinds of things that get labeled as philosophy of science. One kind of thing is the philosophy of how science works. So if you think about Thomas Kuhn, Feyerabend, Popper, the famous names in the philosophy of science, they’re thinking about how we invent theories, how we decide what theories are true, how theories change over time, and it goes right into the sociology of science, different research practices, different communities, how they talk to each other. That is something called philosophy of science.

1:16:57.4 SC: But then there’s something that is usually called philosophy of physics or philosophy of biology, or philosophy of mathematics, okay, things that you might say are part of philosophy of science, but are in fact not interested in how theories are chosen, but they’re interested in how the world works. [chuckle] So a philosopher of physics is interested in the foundations of quantum mechanics or the foundations of spacetime. Philosophers of biology are interested in how evolution works and whether or not there’s selection pressures at different levels or things like that, all that stuff, okay.

1:17:28.0 SC: That kind of philosophy of science is more continuous with science than with the sociology of science, and it’s that kind of philosophy of science that is sort of the philosophical side of science, that is much more what I personally am interested in. But, if you were to buy a book on the philosophy of science that purported to be an overview, I think that most of it would be about the former kind. It would mostly be about theory choice and how research programs work and stuff like that. And that’s interesting and important stuff, but I’m not an expert in it, so I don’t really have any good recommendations there. Whereas if you wanted to buy a book in the other stuff, it’s probably a different book for philosophy of physics versus philosophy of biology versus philosophy of mathematics or something like that.

1:18:15.0 SC: I can think of books off the top of my head in all those areas, although I’m not familiar enough with the entire set of books to say that they’re the best. There’s a set of books by Tim Maudlin that are introductions to philosophies of physics. There’s a new book by David Wallace, a very short introduction to the philosophy of physics, which I haven’t seen, but I’m sure is very good. There’s a book by Alex Rosenberg, previous Mindscape guest, on the philosophy of biology that is very good, an intro book. And the philosophy of math, I have a few books, and I can’t remember, honestly, who wrote them, but you could… I haven’t read any of them, I just own them. I’ve glanced through them for particular reasons, when I needed to know one fact about Gödel’s theorem or whatever. But that’s my overview of the whole state of the field. So, I can’t really offer one or two books that answer the question.

1:19:03.9 SC: Connor Scott says: If a neutron star is propped up by the Pauli exclusion principle, and if the Pauli exclusion principle only forbids fermions from being in the same quantum state, then wouldn’t it be the natural assumption in the case of a neutron star gaining mass and collapsing into a black hole, to say that the simplest ground state of the fermions is no longer to distinguish their quantum states by position, but by some other factor?

1:19:27.5 SC: So, there’s a simple answer, but behind it, there’s a more complicated issue here. The simple answer is, we know what the degrees of freedom are that fermions can have, at least the fermions we know about, like protons and neutrons or the quarks that make them up. So, there aren’t new factors that can distinguish between them. Otherwise, there wouldn’t be neutron stars, because you could just pack different particles with different values of those degrees of freedom as densely as you want.

1:19:51.8 SC: So, it’s an interesting fact. It goes into… I wrote about this… We talked about panpsychism, or we didn’t talk about panpsychism, but I’ve been thinking about… We talked about consciousness. I remember, we talked about Philip Goff. That’s what it was, someone mentioned panpsychism and Philip Goff. So, Philip is having a… Is editing a collection of essays in response to his book called Galileo’s Error, and essays from different people from very different perspectives. And I was one of the people he asked to write an essay. So, I did that and those of you who follow me on Twitter might have seen it, called Consciousness and the Laws of Physics.

1:20:33.1 SC: And of course, there’s a whole bunch of things that have been written about the consciousness and the laws of physics, and my thing is different because I’m really not talking about consciousness at all. In the same spirit as my book, The Big Picture, I’m just trying to make the argument that whatever consciousness is, it’s probably not a good idea to try to explain it by changing the laws of physics. And this does get back to the Pauli exclusion principle, just trust me.

1:20:52.4 SC: The connection is that one way that you might imagine to be panpsychist, one way and only one way, not the only way, but one way to imagine being panpsychist is to imagine there is a new degree of freedom for material particles like electrons and quarks, namely, a mental degree of freedom. There might be two mental states, just to keep things simple, maybe there’s a million mental states. But there might be two different mental states that an electron could be in, maybe happy or sad. But the point is that if that new mental degree of freedom acted like an ordinary physical degree of freedom, like spin, for example, we would have detected it a long time ago.

1:21:34.5 SC: The number of degrees of freedom that elementary particles have comes into calculations like the probability that two electron scatter off of each other or something like that, okay. So anyway, we know how many degrees of freedom particles have, there aren’t any new ones. And that’s true for new possible mental degrees of freedom, but it’s also true for possible new physical degrees of freedom. Now, what is possible is that the neutrons and the neutron star can, under great pressure, convert into even heavier particles.

1:22:04.5 SC: The whole reason why when a white dwarf collapses the electrons and protons convert into neutrons is that the neutron is heavier than an electron, and therefore you can squeeze them more tightly into each other. So, probably what happens along the way of the collapse of a neutron star, is that neutrons sort of become energetic enough so they can convert in to even heavier particles and be squeezed more closely. But as far as we know, whatever they convert into, it’s not a stable configuration that would stop them from collapsing all the way into a black hole. As far as we know.

1:22:38.9 SC: Chris Shipton says: Is there a relationship between the halting problem and the traveling salesman problem?

1:22:47.5 SC: Roughly speaking, no. I’m not a super expert at this stuff. So, of course there are some relationships in that they are both problems in computational complexity theory in some sense. But the traveling salesman problem is hard, it takes a lot of steps to figure out what is the best route between all these different cities, but the halting problem is literally unsolvable. There’s a difference between unsolvable and hard. Hard means it takes a lot of steps.

1:23:14.1 SC: The halting problem is it literally can’t be solved. There’s literally no way of knowing just by looking at a computer program whether it is guaranteed to halt or not, other than running it and seeing if it halts, but there you can only show that it does halt in a certain period of time, you can’t show that it doesn’t. If you run it for an hour, you’re not… And it doesn’t halt, you’re not sure whether it would run for a year or run forever. So, that’s a literally unsolvable problem, which is very different than the traveling salesman.

1:23:42.5 SC: John Dick says: Given an Everettian perspective, what would be the consequence if our universe, the vacuum, happened to be in a metastable state with a decay time of only a million years? Could we tell?

1:23:54.9 SC: The Everettian perspective doesn’t change anything in questions like this. It’s the same perspective, it’s the same quantum mechanical prediction. And the prediction is that if we’re in a metastable vacuum, there’s a probability per unit time and per unit spatial volume that a lower energy vacuum tunnels into existence. So, if the decay time for that happening is a million years, you need to tell me per what volume also, but I get what you mean. Let’s say per the volume of our observable universe, okay. Well, our observable universe is much more than a million years old, so it would have been overwhelmingly likely that we would have already decayed.

1:24:30.6 SC: Now, there is an entropic cutoff, there’s an entropic consideration. Maybe it is the case that if the new vacuum nucleates and takes over our old space, the laws of physics within the new vacuum do not allow for the existence of life, and therefore there is one branch of the wave function where these decays haven’t happened yet, and that’s the only place we can live. I can buy that, but then we would, within a million years, we would probably die. That would be the prediction of that. And there’d be some other branch of the wave function where we hadn’t died yet, etcetera, etcetera.

1:25:07.5 SC: So, I think that that’s a little fishy. I’m not a fan of those kinds of anthropic arguments, ’cause I don’t know what the laws of physics are going to be in the new universe, etcetera, etcetera, and it’s just much easier in the space of all my credences to imagine that the lifetime of our universe is really, really long, billions of years, and therefore it was just likely we haven’t seen it yet. That’s my guess.

1:25:30.3 SC: Anders says: The Kalam cosmological argument for God is regularly trotted out. Everything that begins to exist has a cause, the universe began to exist, therefore the universe has a cause. And with a little hand-waving you get to God. This seems like something a cosmologist would have something to say about, is there a reason why you don’t accept it?

1:25:47.7 SC: So, yes, for some of you, you’ll know that I did a debate with William Lane Craig about this very issue, ’cause this is William Lane Craig’s favorite proof for the existence of God. And the short version to my response is, we don’t know whether the universe began to exist or not. Maybe it did, maybe it didn’t, that’s just an open question. But we do know that the first premise, everything that begins to exist has a cause, is just false. That’s just not the right way to talk about fundamental physics kinds of questions. There’s two options that you have. One is, if you’re in the emergent higher level world of me and you, and tables and chairs, and cups of coffee, and things like that, then there is a very natural language of cause and effect, and that language of cause and effect is very dependent on entropy and the arrow of time, okay?

1:26:34.3 SC: And so, we’re not describing the world at the microphysical level, we’re not Laplace’s demon, we have incomplete information about the world, we coarse-grain the world, and in that higher level coarse-grain description causes and effects are very, very important. The other option, but obviously, it doesn’t tell us anything about the beginning of the universe ’cause that is not described by that course-grained higher level description.

1:26:55.5 SC: The other option is that we give a description in the microphysical language, the fundamental laws of physics, okay? But as far as we currently understand the fundamental laws of physics, they don’t have cause and effect in them as concepts. Instead, they have differential equations, and they say the universe, whatever mathematical structure you are using to describe it, like a wave function or something like that, obeys this equation. That’s the laws of physics. There’s not an extra law that says that things have causes, okay? The only law is, “Here’s the stuff, it obeys this equation.” That is our current paradigm for doing the laws of physics.

1:27:33.2 SC: And so, the point is you don’t say everything that begins to exist has a cause, you say everything obeys the laws of physics. And once you say it that way, you’re not going to get to God existing. You just say, “The universe obeys the laws of physics, the universe may or may not have begun to exist.” [chuckle] And there’s no therefore, God exists. That’s just not a possible conclusion.

1:27:56.8 SC: Siddhartha says: Can any seemingly non-deterministic universe be truly random at the fundamental level? Even if entities inside the universe cannot tell, doesn’t the universe itself somehow need to pick one or all, in many worlds, a possibility following some deterministic process?

1:28:17.9 SC: So I think, Siddhartha, you’re just sort of insisting that you don’t believe that things can be non-deterministic. And I don’t know what to tell you, I don’t think we have the right to make those insistences. It’s a nice feature of the laws of physics in our current best understanding that they are deterministic, but if we learn more tomorrow and learn that they weren’t, then we would have to accept that. I see no reason whatsoever to say a priori that I can’t imagine non-deterministic laws of physics. I mean, that’s just a failure of your imagination. I can imagine non-deterministic laws of physics, no problem. I don’t know what do you mean by imagine here, but I can certainly write down possible worlds that obey non-deterministic laws. And we could live in one of those. That’s just an empirical question. I don’t know what else to say.

1:29:04.9 SC: Corbi Ziesman asks a priority question: My question ties into the quantum suicide-type thought experiment, but is related to blockchain security. Basically, your cryptocurrency money is protected by a 256-bit private key, nearly impossible to guess, but one could use a quantum random number generator, or the Universe Splitter app. To generate 265 one-bit quantum results. This means you would have 2 to the power 256 timelines in superposition, where each version of you tries a unique 256-bit permutation. And this exercise is all possible cryptocurrency private keys, meaning there is a timeline where you successfully guessed every single user’s private key. And then it goes on a little bit further than that.

1:29:47.7 SC: But I can answer the question here, because this is an answer that I give to many similar questions. This is a particular version of this question where it’s been dressed up with cryptocurrency, or blockchain, or whatever, but it’s a version of the following question. There is a set of things which in traditional ways of thinking about quantum mechanics are just stochastic, are just unpredictable. And in many worlds, you predict with 100% probability that there is a world in which one answer comes true, and a world in which another answer comes true, and then some attempt is made to leverage that into some important difference.

1:30:21.4 SC: And I’m going to say it again and again, there is no important difference between those cases. By important I mean for us human beings. To say that guessing something with a chance of being correct of 2 to the minus 256, is to me exactly the same, for all intents and purposes, as saying there will be 2 to the power of 256 universes in one of which I guessed correctly and the other one in which I guessed wrong. For all practical purposes, for all actionable, pragmatic purposes, how should I behave in this world, those two situations are exactly the same.

1:31:01.7 SC: The fact that in one out of the 2 to the power 256 universes there is a version of me that got it right means nothing to me. It’s exactly the same as saying that there was a 1 in 2 to the minus… One in 2 to the 256 chance that it would have gotten it right, that’s what it means. So, I don’t think that that matters in any real tangible sense.

1:31:27.1 SC: Allison says: Could quantum networking be used to build a telescope that spans planets?

1:31:32.5 SC: So, no is the simple answer here. But let me just very quickly elaborate on this idea of quantum networking, namely, there’s no such thing, at least in the sense in which you probably mean it. Remember, there’s entanglement between particles that can be separated by a large distance, but you can’t use entanglement to send information. You can’t use entanglement to, I don’t know, run a telescope or something like that. Entanglement is not a tangible connection. If you want, and there’s different ways of thinking about it, of course, there’s different ways of conceptualizing it, but if you want, entanglement is a prediction. Entanglement is a prediction for future experimental outcomes.

1:32:11.7 SC: It’s a prediction that if I measure one particle in one state, then when I go over to visit the other particle, it will be in another state, if those two particles were entangled with each other. And that kind of fact about predicting the future is just not very useful for building a telescope. So, maybe you can build a telescope that spans planets. That’s not to say that’s impossible. But the way it will work is you build some instrument on each planet and then you just send the information that one gets to the other one. And quantum mechanics has nothing to do with it.

1:32:44.0 SC: Stefan Berninger says: “I very much liked your conversation with Stephen Wolfram. He has not yet shown that his approach can actually explain our universe, but at least so far, there’s nothing disproving his approach. I am, however, not sure how he will manage to overcome the many orders of magnitude between the scale which is reachable by computations and the scale where observable physics starts. I am thus curious to hear your main takeaways from this conversation, and if it’s somehow changed your view on how to address the open questions on a most fundamental level.

1:33:12.3 SC: So no, I don’t think it’s changed my view very much. I think that the respectable way to think about what Wolfram is doing is, he’s crossing his fingers. [chuckle] He’s saying that there is a chance… What he’s trying to do is guess the fundamental laws of physics, starting from almost nothing. Starting from, well, there’s some things and there’s some rules. That’s basically the starting point. Now, maybe he’ll get it right, it’s possible. It’s possible that you can guess the laws of physics. It might involve some trial and error along the way, but my experience as a scientist is that it’s not going to be that easy. The world surprises us, the world that when we observe it, when we measure it, when we interact with it empirically, it does things that we wouldn’t have guessed ahead of time.

1:34:04.9 SC: In particular, we have quantum mechanics. And I think that there’s only two options for Wolfram’s project. One is, he recovers quantum mechanics exactly, and the other is he doesn’t. Those are the two possibilities. If he doesn’t, then that’s very, very exciting because he should be able to make a prediction that deviates from the predictions of quantum mechanics, and that would be testable and that would be great. And it’ll either work or it won’t. But if he does get quantum mechanics exactly, then I… To me, why not start with quantum mechanics? Why are you trying to guess some rules that will get you to quantum mechanics? It would be fun to discover some even more primitive rules that get you to quantum mechanics, but my project is much more interested in getting from quantum mechanics to the observable world.

1:34:53.4 SC: So, to get from almost nothing to quantum mechanics is fun and interesting, but not the way that I am personally interested in pursuing physics. Not to say that he shouldn’t do it or other people shouldn’t do it, it’s just my personal view.

1:35:08.4 SC: Brad Malt says: “In 1968, track and field star Bob Beamon broke the world record for a long jump by almost two feet. This is a margin so big that it was named one of the five greatest sports moments in the 20th century by Sports Illustrated, and remains an Olympic record over 50 years later. Do you think this amazing jump could have been an instance of the wave function collapsing to a very unlikely event, the sports equivalent of putting one’s fist through a solid table? Before you say this is too unlikely to expect it to have happened, consider that the relevant universe of events may not only… Be only long jumps in the Summer Olympics, but something much larger, like all athletic events in the Summer Olympics or all athletic events in the history of the universe or all muscle contractions in the history of the universe.

1:35:49.8 SC: Okay. So I think, Brad, that this is a very typical mistake, honestly, a mistaken way of reasoning. So you’re saying that, is it possible that this particular unlikely event was caused by this other unlikely event? And you’re saying, well, on the one hand, sure, an individual case of this unlikely event is very unlikely, but on the other hand, the number of times that we’ve done events is so large, so even an unlikely event is bound to happen eventually. The clear mistake in that kind of reasoning is, it depends on how unlikely the unlikely event is. No matter how many, how large the space of all events is, I can pick an unlikeliness so that none of those events are likely to happen, none of these unlikely events are likely to be one of these examples.

1:36:38.3 SC: The obvious comparison is with the existence of intelligent technological civilizations on other planets. There’s a lot of planets in the observable universe, some huge number of them, and so some people try to say, “Well, they must have intelligent alien life on them because there’s so many planets.” Well, you don’t know because you don’t know the probability of there being other planets, of intelligent life on any other planet. If the probability is 10 to the minus 100, then there’s probably not intelligent life anywhere else in our observable universe.

1:37:08.9 SC: Likewise, you can’t just say, there were a lot of athletic events in the history of the universe or a lot of muscle contractions. You have to compare it to the quantum probability of something really dramatic happening on this macroscopic scale. So just to illustrate this, let’s go through it, let’s go through this in our heads in real-time. Let’s imagine… Let’s be somewhat generous. I don’t know how to estimate the number of muscle contractions in the history of the universe, but let’s imagine the number of jumps that have been done by human beings in the history of the universe.

1:37:39.0 SC: And again, let’s overestimate it so that we’re safe, okay? So, there’s been about 100 billion human beings alive on Earth throughout history, very roughly speaking. The details are not going to matter here, trust me. The specific numbers are not going to matter here. So, 100 billion, that’s 10 to the power 11, okay, human beings in the history of the universe. And let’s say they jump once every three seconds. Most human beings don’t do that, but again, we’re over-estimating. So, there are 3 times 10 to the 7 seconds in a year. So, if a human being jumps once every three seconds, that is 10 to the 7 jumps per year per person. So if there are 10 to the… That’s right, yeah.

1:38:25.8 SC: But you live for 100 years. So for a person’s lifetime, it’s 10 to the 9 jumps per person’s lifetime, and there’s been 10 to the 11 people, so that’s 10 to the 20. That’s how many jumps people have done in history. Now again, I don’t know about you, but I don’t jump once every second, but still, we’re over-estimating. So the true statement is there are equal to or less than 10 to the 20 jumps being done in human history. That’s a big number, right? It sounds like a lot, maybe one of them could have been a macroscopic quantum event. Really, really, really unlikely, but it happens once in a while. Clearly, the number of competition long jumps or competition jumps of any sort is way smaller than 10 to the 20, but still… Let’s stick with 10 to the 20.

1:39:10.5 SC: How likely is it that a macroscopic quantum fluctuation was relevant to that jump? Well, the bad news is that, roughly speaking, the likelihood of a macroscopic quantum fluctuation is exponentially small in the number of particles that you’re talking about fluctuating. So, this is not quite fair, but it paints a vivid picture here. Let’s say you need a gram of matter to fluctuate in your muscle to make this relevant. You probably need more than that, but let’s imagine it’s that much. So, that’s roughly speaking, again, very, very roughly, Avogadro’s number of particles in a gram of matter. Avogadro’s number, let’s call it 10 to the 23. It’s more like six times 10 to 23, but we’re rounding here and we’re rounding down in this case.

1:40:00.1 SC: So, 10 to the 23 particles need to be involved in this quantum fluctuation, but that doesn’t mean that the probability of the quantum fluctuation is 10 to the minus 23. It means the probability of the quantum fluctuation is roughly 10 to the minus 10 to the 23, okay? That’s why you don’t see large quantum fluctuations in the world. That’s why you say, “Sure, I can bounce a ball off the wall, maybe it will quantum tunnel through, it’s unlikely to do it.” But the point is, it’s really unlikely to do it. So, we’re taking the large number, 10 to the 20, the total number of jumps done by human beings, times the likelihood, the probability that one of them is a macroscopically important quantum fluctuation, but that probability is 10 to the minus 10 to the 23. So, the answer is 10 to the power 20 minus [chuckle] 10 to the 23.

1:41:00.8 SC: So, it’s a very, very, very, very, very, very tiny number. That’s why you can’t just say things happen a lot, therefore, there should be big quantum fluctuations, quantum fluctuations are really unlikely. And I know that for those of you who are experts in quantum mechanics, I’m doing a very, very sloppy estimate for the likelihood of a quantum fluctuation, we haven’t said what kind of fluctuation it is, etcetera, etcetera, but the point stands, the probability of big quantum fluctuations is really very tiny.

1:41:27.3 SC: Okay, Stephen Bernard says: You’re invited to host a panel discussion of the greatest physicists of all time, living or dead, language is no barrier, but you’re limited to three. Who would you pick?

1:41:38.8 SC: So I don’t… Yeah, I mean, part of me wants to say, “I don’t care.” I don’t like these kinds of thought experiments because there’s an implicit idea in these thought experiments that there is some wisdom that these people have that I would like access to, and I don’t think that’s true. I don’t think that there is any intrinsic, more intelligence on the part of these dead people than there are on the part of currently living people, and I can talk to the world’s best physicists who are alive right now. I’ve had some of them on my podcast, [chuckle] so what’s the point?

1:42:13.2 SC: And the other thing besides intrinsic intelligence is of course knowledge, but we have way more knowledge than those folks have. I’ve said it before, and it’s not bragging, but I know a lot more about general relativity than Einstein ever did. That doesn’t mean that I’m smarter than Einstein. He invented it, but we made progress, and he died many decades ago. Einstein didn’t know about black holes at all. He certainly didn’t know about singularity theorems or anything like that. He had a very fuzzy knowledge of gravitational waves, of cosmology, and other things. So, Einstein doesn’t have a lot to teach me about general relativity, is the truth of it.

1:42:50.9 SC: Nevertheless, the reason why I did answer the question is because what you might want is history, that’s what you don’t have, history or biography or something like that. So, really what you’re asking yourself is, who are the people, who are the persons, who are the characters you would have liked to learn something from about what was going on at that time. Even given that caveat, I really don’t know what the answer is. So, just to pick some people… I think Galileo would be a good one. He led a colorful life and Galileo in many ways really pre-figured modern science. He had a way of thinking that was super duper ahead of his time that I think is very interesting, so I would have liked to talk to him about that.

1:43:35.5 SC: Aristotle would be interesting because of lots of reasons, but maybe someone like Democritus or Epicurus or Lucretius, one of the early atomists, just because we don’t know a lot about what their discussions were like back then. There’s not a lot of writings that survived, so that would be interesting. And I guess the other one would be Ludwig Boltzmann, because he wrestled with these deep questions about probability and the nature of statistical mechanics, and sometimes he said things that were brilliant and very far-seeing, and other times he missed things I thought were pretty obvious. So, I would like to know what his thought process was. But these are all history questions, I’m not expecting to learn anything about physics or philosophy from these folks.

1:44:17.4 SC: Peter Benham says: I know you have a distaste for ‘it seems’, but it seems logically obvious that it had to be possible at the Big Bang for emergent phenomena of high complexity like life and consciousness to evolve, though it would seem impossible that one would have conceived of such things by just looking around at the raw materials, and yet here we are. What are potential higher order emergent phenomena that seem impossible that we might be able to conceive of emerging eventually somewhere, somehow?

1:44:46.0 SC: Yeah, so I don’t know, I don’t know. You’re right, I don’t like this, ‘it seems.’ Like the previous question about the quantum fluctuations, you have to work things out. It’s just not good enough to say it seems possible. There’s a probability of things happening and you have to sit down and do it. That’s why science is hard, that’s why it’s not just BSing around the coffee table. So, I don’t know exactly what is meant in this case, by phenomena of high complexity to evolve. Presumably you mean that it was not possible for life to evolve near the moment of the Big Bang, but that the possibility of life was inherent in the materials and laws that were there at the Big Bang. It was possible that billions of years later things would evolve into us. Yes, which I agree with that.

1:45:34.3 SC: But anyway, with all that caveatting out of the way, the reason… I’m not going to give you a good answer to this question, but I can try to explain why it’s hard to give a good reason. The biggest reason why it’s hard is ’cause we don’t have that many examples, empirically, of complex information processing, higher order emergent phenomena. We have life here on Earth, that’s basically it. Which is not to say that’s the only thing that could have happened, but the only way that interesting… If you’re interested in sort of the level of complexity that you would begin to characterize as alive or information processing or conscious, we only have one example, roughly speaking. So, it’s very hard to draw grand conclusions.

1:46:20.3 SC: There is one reason to be pessimistic, that there are very different kinds of higher order emergent phenomena that would have that level of complexity, namely that the universe doesn’t last forever. There’s a competition going on between the emergence of life and complexity and the heat death of the universe. Being a conscious creature takes time. There are processes that go on in our brains that take some amount of resources, and one of those resources is duration, is time. So it took, like we said, billions of years to get from single cell organisms to us and the universe is 14 billion years old since the Big Bang. So, that’s an appreciable fraction of the entire age of the universe.

1:47:09.3 SC: And if you think about stars burning, lighting up the universe, the rate of star formation… The universe already peaked in star formation. The number of stars forming in the universe is on the decline now. Now, individual stars can last a long time if they’re smaller stars. They can last a 1000 billion years or even longer than that, but they’re very dim. The smaller stars last longer, but are dimmer, so they have less room, but also a factor of a 1000 is nothing in this game. So, you can imagine kinds of higher order emergent phenomena on very different scales, like the galaxy becoming alive.

1:47:54.1 SC: Fred Hoyle wrote a science fiction book about a nebula coming alive. You could imagine replacing the sort of energy source that for us is the sun with Hawking radiation from a black hole, which is much thinner and cooler, but maybe if you increase the time scales appropriately, something could happen. But you would have to increase the time scales by an enormous amount, and the universe might just not last that long in a very realistic sense. All the interesting stuff might die away, everything might fall into a big black hole and evaporate away, and there’s nothing outside the black hole to be fueled by the Hawking radiation.

1:48:31.9 SC: So, it actually is kinda tricky to get the time scales to work out to give you enough time to not only have a thinking creature, but to evolve that thinking creature through the process of something like natural selection. And therefore, at the end of the day, it’s at least conceivable that the other possible kinds of thinking, conscious creatures in the universe are more or less what we think of as organic life. They could be very different than us literally here on Earth, and yeah, they might even use different DNA, base pairs and stuff like that, but I think the basic chemistry is going to be the same. I don’t think you’re going to see living beings made out of photons or made out of interstellar gas or anything like that. Again, could be wrong, but probably by the time… For my lifetime, we’re not going to discover it, let’s put it that way.

1:49:24.9 SC: Crather Luca says: What exactly does it mean to be a moral constructivist? Is it really different from being a moral realist? For example, people often object to moral realism by stating that there can be no foundation for moral knowledge to be built up from. However, when it comes to physics and mathematics, people are far more willing to accept we are exploring some objective reality rather than merely constructing it, but in physics or mathematics, there can be no foundation for knowledge, either we could be tricked by a demon, there’s Gödel’s incompleteness theorem, etcetera. If we can’t justify our beliefs in areas like math and science more than in morality, then can we say that lacking a foundation is not an impediment to realism? Because now there is a dichotomy between moral knowledge on the one hand than scientific and mathematical knowledge on the other hand, even though they all lack a foundation.

1:50:06.8 SC: So, partly I wanted to read this question because it’s a good advertisement for the podcast episode I did on exactly this topic with Justin Clarke-Doane, who draws connections between the status of objectivity and realism in morality and mathematics. So, I would distinguish… And I should say ahead of time, I’m not a super expert at this either, I know a little bit, but this is not an area where my own points of view are sufficiently knowledgeable that they have hardened into convictions. I have feelings, but they’re very willing to be changed if someone gives me a good argument one way or the other. But what my feelings are, is that science and math and morality are three very different areas. They’re not parallel to each other very much at all.

1:50:56.4 SC: In math, you start with some axioms and then you prove theorems. Now, even that is… Even that is a contentious statement, that’s what follows from Gödel’s theorem. That there could be true statements that are not derivable from a set of axioms in a formal system, but okay, let’s put that aside. The point is that I can have different mathematical systems, whether or not you were thinking of them axiomatically, or some other way. I can have Euclidean geometry, non-Euclidean geometry, I can have different but equally consistent sets of axioms for set theory, etcetera, okay?

1:51:29.3 SC: Whereas in physics or in science, there’s the world, there’s the real world and there’s one real world, rhetoric of the multiverse, etcetera, aside, count that all as one real world. There’s one set of stuff doing something that could have been otherwise, right? So, unlike math where you say, Well, there’s some foundational principles, axioms or something equivalent to that, and different possible systems you can build on that, there’s different possible physical systems, but there’s one real one. As I said before, I’m not a modal realist. I think there really is something real about the individual real world in which we live.

1:52:04.9 SC: Now, it’s not foundational. I agree with you that we could be being tricked by a demon, or we could be Boltzmann brains or whatever, but science progresses under the assumption, the hypothesis that that’s not true, that there is some basic relationship that is not too tricksy between our sensory data and the real world. It’s not immediate, we can be tricked, there are optical illusions and so forth, but we’re not being tricked in some intentional way by an evil demon or something like that, and therefore we can start with our sense data and build up a picture of the real world, we just have to assume that we’re not being intentionally misled along the way.

1:52:49.7 SC: But it’s a very different thing than math. In math, we imagine different possible mathematical structures, and they’re all very conceivable. In science, there is many different possible worlds, but only one of them is real. In morality, it’s completely different. In morality, neither one of those is true. It’s not that you have axioms and you prove theorems and that’s it, nor is it true that there’s one right system and you’re looking for it, that’s the essence of being a constructivist. Morality to a constructivist is a way of taking our built-in moral impulses, let’s say… Which, and I say built-in, but that… Don’t let that mislead you, it could be that they’re in part instinctive, but it also could be in part that you learn them growing up and they change over time, etcetera, but we all have moral impulses, we all have things that we think of as right or wrong.

1:53:41.0 SC: And to a moral constructivist, what you think of as morality is just a systematization and a rationalizing of those moral impulses into a set of beliefs, which we call morality, and different people might have different impulses, so unlike physics, where there’s only one real world, different people might be very respectably and correctly led to different moral conclusions, and then the moral constructivist says, talk it out, sit down, talk to each other, try to persuade each other because the whole point is that our sort of starting point moral impulses are not coherent, they don’t necessarily fit together, they’re not even necessarily compatible with each other, we need to think about them, and we might realize that, Oh, let’s downgrade this one, and let’s upgrade this one, let’s invent this new one, it’s constantly a set of flux. And so, there’s not only no foundation, there’s not even any reason to agree between different people for a moral constructivist. That doesn’t mean that I need to believe and accept what other people think is morality, that’s the difference between constructivism and relativism.

1:54:47.9 SC: I can say, I think I’m right and you’re wrong, but I don’t think I’m objectively right. I think that I’m right, and I’m going to try to convince you that I’m right, but there’s no way that I can say you’re making a logical mistake if I don’t convince you of that. Whereas in math, I can say that you’ve made a mistake proving your theorem and in physics, I can say your theory about how the world works is wrong because of this experiment. There’s no such thing that we have access to in morality.

1:55:13.1 SC: Okay, I’m going to group two questions together, because they’re about photons and massless particles. Alexander Cordova says: How is it that massless particles like photons can have non-zero momentum? I’ve worked through a special relativity chapter of an undergrad physics textbook, and there’s an equation E squared equals MC squared squared plus PC squared. That is to say energy squared, let’s set the speed light equal to 1. So then, energy squared is mass squared plus momentum squared, and then the same text book, he says, also gives the following equation for relativistic momentum, P equals MV divided by the square root of 1 minus V squared. And he says, no matter how I rearrange these equations or try to plug in various equivalent values, if I plug in a rest mass of zero, I get a momentum also equaling zero. So…

1:56:03.0 SC: And the other question… I’m sorry, I’m not grouping together another question, I’m mentioning an answer. So Thomas Prunty gives an answer that is half correct and half I disagree with. So let me say it. So what Thomas says is, if you put V equals C and M equals zero into the relativistic expression, you get zero divided by zero, so in other words, it’s undefined. So Alexander got a misleading answer that the momentum must be zero, by first plugging in mass equals zero and then getting that the answer was zero, but not at the same time plugging in the velocity was equal to the speed of light. If you do both of those at the same time, you get zero divided by zero, it’s undefined. And I think the right way of saying that is that, that equation, momentum is MV divided by the square to 1 minus V Squared, just doesn’t apply to massless particles. That’s supposed to be an equation that only applies when the mass is not zero.

1:56:55.5 SC: The other equation, E squared is M squared plus P squared, always applies. That one is always true for any kind of particle. But then Thomas goes to ruin it by saying, a photon is a quantum mechanical object, and so you don’t need… So you do need quantum mechanics to get its momentum, relativity doesn’t have an opinion on the matter. That, I have to take issue with, because even though it’s true that photons are quantum mechanical objects, relativity certainly does have an opinion on the matter. Within relativity, you’re certainly welcome to talk about particles with zero… That move at the speed of light, massless particles, particles with zero mass. In relativity is where you get the answer, that particles that have zero mass will move at the speed of light and their energy will be equal to their momentum squared. Sorry, the energy squared is the momentum squared, so E equals P is the relationship between energy and momentum for massless particles, and that’s a perfectly classical statement, there’s no obstacle in principle to talk about classical massless particles in relativity. It so it happens that the real world, and therefore photons are quantum mechanical, but relativity would work perfectly well anyway.

1:58:04.3 SC: Okay, Tom Kwine gives us a priority question: You wrote in From Eternity to Here, that entropy quite literally makes life possible. I have an inkling that entropy is the motor of evolution, the driving force that requires things to either persist or to perish. Can you make a hypothetical argument for why this might be true, and a parallel argument for why it might be the wrong way to look at things?

1:58:27.9 SC: No, I can only give you an argument for why it’s true, ’cause it is true, but we need to be careful about the sense in which it is true, okay. The statement that entropy quite literally makes life possible can be justified by saying the following thing. What if there were no increasing entropy in the universe? What if the universe was just in equilibrium? What if the universe didn’t have an entropy going up at all? Then nothing would happen in equilibrium, like if you have a gas in a box and it’s all in one corner and it’s low entropy, and then you let it go and it fills the box, it reaches equilibrium and then it stops evolving.

1:59:04.7 SC: Life is a process, as you already said, life requires dissipation into the universe, and dissipation means increasing the entropy of things around it. Life is time-directed, life is not like a pendulum going back and forth, life remembers the past and doesn’t remember the future and so forth. So to do all that stuff that life does, entropy does make it possible, but the particular aspect of entropy that makes it possible is that it is possible for entropy to increase. If you are at thermal equilibrium, where entropy could not increase, then life could not exist, ’cause you wouldn’t have any of that rich set of irreversible processes going on.

1:59:47.8 SC: Having said to that, you can oversell the case. When you say entropy is the motor of evolution, yeah, I would say that entropy is a necessary condition for evolution, that doesn’t quite mean the same thing as it’s the motor of it. After all, there’s plenty of cases where entropy increases without evolution going on, without life existing at all. Life is a particular kind of complicated thing that requires not only entropy to increase, but other things as well. Hang in there for a recent… For an upcoming podcast, where we’ll talk about entropy and the arrow time in interesting ways.

2:00:28.1 SC: Douglas Albrecht says: Infinity and infinitesimals seem to create challenges in physics, and it seems that they require what seem like work-arounds. Energies at infinitely small distances require cutoffs, cosmology seems to blow up the beginning of time, etcetera. Quantizing spacetime also seems to be another way of dealing with this. So I wonder if maybe it’s the math that needs evolution to deal with limits, or we are just not equipped to deal with these extreme points.

2:00:53.8 SC: So I don’t think it’s the math’s fault, maybe it is, it’s certainly a possible idea. So think about the infinities of quantum field theory. If the general problem we’re facing here is, there are infinite or undefined quantities in physics and we need better math to understand them, the infinities in quantum field theory are an example, where in some sense, better math came along and helped us understand what was going on. Feynman, Schwinger, Tomonaga, Dyson, those people showed us how to re-normalize quantum field theory, which is basically a fancy pants way of saying, “Take a limit,” where you have some function and you’re just trying… You can’t evaluate it at a point, but you can take a limit as it gets closer and closer to that point and find the same answer. So they figured out a way to do that in the case of quantum field theory, by using some fancy math.

2:01:42.2 SC: On the other hand, we now know that there’s an even better way of thinking about the infinities of quantum field theory, which is the Wilsonian way. Ken Wilson showed that if you just take seriously the fact that you don’t know what’s happening at very, very high energies, very, very short distances, you can still do quantum field theory, you can do effective quantum field theory. And honestly, all of these infinities came about because you were just taking infinite energies and zero wavelengths too seriously. What right did you have to do that? Probably spacetime itself could potentially break down at those points. So Wilson said that the existence of the infinities in the first place was a mistake. And so in some sense you didn’t need fancier math to deal with them.

2:02:28.2 SC: And I think that that’s more along my guess as to how things are going to go, that it’s not that you’re going to need fancier math to deal with all these things, it’s that you’re going to realize that the underlying physics is a little bit simpler or a little bit less dramatic than it seemed to be. But of course, there’s different situations here. We’ll have to see how it goes.

2:02:48.4 SC: Okay, I’m going to group three questions together about black holes and information, about unitarity. Matt Hickman says: From my understanding, one of the guiding principles used to think about black hole information is that information is not destroyed in quantum mechanics. If I were a believer in the Copenhagen interpretation, I might say that information is destroyed all the time, a wave function happily evolves according to the Schrödinger equation, and then you measure it, and information is gone.

2:03:14.5 SC: Humberto Nani says: May you please comment on what is lost if unitarity is lost, like which parts of the current theories would be affected. And Rich Hogg says: Hawking radiation is usually described as emerging in a specific thermally randomized configuration, but under the many worlds interpretation, wouldn’t we more correctly think of it as merging into a superposition of all possible configurations, which then decohere into specific configurations in separate branches?

2:03:40.5 SC: So all of these… So let me comment on Humberto’s question first, because I want to just define what is going on. The question is, “What is lost if unitarity is lost?” So unitary is just the fancy physicist term for obeying the Schrödinger equation, roughly speaking. If you think about what the Schrödinger equation tells you, if you have some wave function, some quantum state, the Schrödinger equation is deterministic, it evolves one wave function, given some Hamiltonian, AKA, some laws of physics, it evolves the wave function over time in a perfectly predictable way.

2:04:13.3 SC: And famously or infamously, in quantum mechanics, that’s half the story. The other half of the story is what happens when you measure a quantum mechanical system and then it’s not unitary, the wave function collapses, and it’s unpredictable. It’s not deterministic. So when we talk about unitarity being important in physics, it is implicit, that we’re talking about the part of evolution where we’re not measuring it. So whenever physicists worry about the black hole information loss problem, they go, “Ah, unitarity, blah blah, blah.” They all know the unitary is violated when you make a measurement. But they’re just not saying that because they’re talking about a system as a black hole, it’s out there just doing its thing. It’s not being measured. That’s the implicit idea.

2:04:58.9 SC: Now, I’m not sure if that’s okay. I think that physicists, because they don’t fret about the foundations of quantum mechanics, tend to be sloppy about these issues, and they sometimes will say, “Oh, you must be unitary,” and other times you will say, “Well, of course, you’re not unitary when you measure and etcetera,” and they don’t tell you exactly what is meant by a measurement, etcetera, etcetera, etcetera. So I think we should worry about that a little bit. To directly answer Humberto’s question, if the fundamental laws were not unitary, so if Schrödinger’s equation was not correct, that’s different, that’s a much more radical thing than saying that when you measure, it’s not unitary.

2:05:38.9 SC: Because we all know, from things like the many worlds interpretation, that you can have a theory that is overall perfectly unitary and yet effectively within observable branches, it looks non-unitary. So if you had really fundamental laws of physics that were not unitary, well, then that would depend. It would depend on how you didn’t have unitary. It could go very wrong, very, very quickly. There were the papers, years ago, that said energy conservation could be dramatically violated if you violated unitarity. All of the tests we’ve done on precision electroweak measurements and things like that in particle physics could be very, very wrong.

2:06:18.5 SC: But then there are other ways you could be non-unitary that are okay. There are theories, like objective collapse theories of quantum mechanics, Penrose, GRW, etcetera, which do violate unitarity in that sense. So it depends. There’s only one way to be unitary, there’s an infinite number of ways to be non-unitary. So it depends, is the answer.

2:06:36.8 SC: Now, to Matt’s question: What about the fact that information is destroyed all the time? And to Rich’s question: In many worlds, you evolve into all possible configurations and then decohere.

2:06:51.5 SC: Yes, the implicit ideas in both of your questions are correct, but I think you need to be careful here, and in fact, I’m sufficiently convinced that you need to be careful, that I wrote a paper about it. So I wrote a paper, well, I co-authored a paper, called Black Hole… What was the name of my paper? Branches of the Wave Function Need Not Contain Black Hole Firewalls, something like that. Our idea was the following, that the firewall argument was a popular thing that people were thinking about, over the last 10 years. It said that you can’t simultaneously have the entanglement of particles, virtual particles escaping from black holes and falling in, near the event horizon, causing Hawking radiation. And get all the information out of a black hole, and therefore they suggested breaking the entanglement near the horizon, which gives rise to a firewall.

2:07:47.6 SC: And what we suggested in my paper and this… I hope I get all the authors here ’cause I was working with a whole bunch of grad students on a whole bunch of different papers at the time. This is with Grant Remmen, Ning Bao, and Aidan Chatwin-Davies, I’m pretty sure. I should really look that up, I’m going to look that up as I’m talking here. But our point was, look, when you say that two things are entangled with each other, two quantum mechanical systems, that statement depends on where you are, in the wave function of the universe. That is to say, on what branch you’re in, in the wave function of the universe. Oh, yes, and Jason Pollock. I knew I was forgetting Jason’s name. So the name of the paper is, Branches of Black Hole Wave Function Need Not Contain Firewalls.

2:08:31.4 SC: So let’s say you have two particles, let’s say they are entangled with each other. Alice and Bob had their two spins, you have two particles and they’re entangled, and then you measure one of them. You’re Alice, you measure your particle, you see it spin up, and now you know the other particle is spin down. What does a many worlds person say? A many worlds person says, “Well, now there’s a branch where the particle… Alice’s particle is spin up, Bob’s is spin down, and another branch where the opposite is true, but on both of those branches, Alice’s particle and Bob’s particle are no longer entangled. They are entangled in the wave function of the universe, because they were before, and you didn’t stop that, but on the individual branches, they’re not.” So that’s the point that we tried to make, and actually, other people have made related points before.

2:09:14.8 SC: But the point is that, if you don’t take seriously the measurement process and decoherence and many worlds, then you can be sloppy about what needs to be entangled with what, because two sub-systems can be entangled in the wave function of the universe, even though they’re not entangled on the branch where you’re measuring it. And so we tried to make the argument there were more than enough degrees of freedom in the black hole, that on every branch of the wave function, you could see the particles near the horizon be entangled and therefore have no firewall, even though in the wave function as a whole, there is enough entanglement between the Hawking radiation at early times and late times, to restore unitarity.

2:09:57.5 SC: Anyway, that’s probably not making any sense if you haven’t read the paper. But the point is, I’m agreeing with the general philosophy of these two questions that, when it comes to careful questions about black hole information being lost, it might pay to be a little bit more precise about what you mean about measurement and unitarity and decoherence and all of those things.

2:10:20.8 SC: Demon Hat says: Would you rather have legs as long as your fingers, or fingers as long as your legs, fully functioning, and why?

2:10:29.2 SC: I think it’s… I think I gotta go with the fingers as long as your legs. Long fingers sounds better than short legs, but the other thing I wanted to say about this question is, if we do imagine different alien life forms, I think that we tend to be a little bit less imaginative than we should be, about how different they could be than us. If you just had fingers as long as your legs for some reason, that evolution gave them to you, I can imagine you’d think of those as perfectly sensible, logical, like you’re just natural. You don’t go, “Oh, man, I wish my fingers were shorter.” But I don’t know what they’re going to be like, especially just with basic things like size. Aliens that we see in movies are normally human-sized, maybe a little bit bigger, a little bit smaller, whatever, but we rarely see intelligent technologically advanced aliens that are 1 millimeter in size or 100 meters in size or 1000 meters in size.

2:11:25.3 SC: I think we should be more open-minded about what these possibilities are. I said earlier in the AMA, intelligent aliens made out of photons or something like that might be implausible, but we could imagine very, very different sizes and shapes than we actually see.

2:11:40.4 SC: Paul Hess says: What is your method to choose which papers to read out of the near infinite deluge? Do you have several passes where you’re forced to skim many papers, then from that set, choose a smaller subset to reread more closely, or do you read papers that others are talking about in your circles?

2:11:57.3 SC: These days, it’s more the latter. But it’s a complicated thing and it’s going to depend on how the style of your research that you’re doing at any one time. It used to be, I really did try to keep up on all the papers that came out in theoretical cosmology and particle physics, at least to the extent of looking at their titles and abstracts. There is a morning mailing that you can get in the email or you can check online from the archive, that tells you in your specific subfields of physics, what are the recent papers. But my own research has gone away from that. So I used to just do sort of the former option that you list, which is look at all the papers, the ones that looked interesting enough, that they’d be relevant to my research, I would print out and and hopefully read at some point. These days, I’m thinking about more long-term things.

2:12:51.1 SC: I’m not chasing ambulances. And the number of people working on the same questions as I am is relatively small, and those papers appear… They’re relatively sparse on the archive, etcetera. So I more rely on people I know and other papers I read, to point me to things that are relevant. This is why I tell high students, when you write papers, it’s good to reference other people’s papers, it’s actually… Not only is it ethically the right thing to do, to put a lot of citations in your papers, but it makes them more discoverable.

2:13:25.8 SC: Sometimes a great way to find papers that are relevant to a particular question, is to go to… There’s different services on the internet, like Inspire is one, for particle physics, where you not only can see what papers are cited by a certain paper, but you can see what papers cite a certain paper. So if you discover a paper is really important to you, then you can see who else has been writing about that, toward the future of when that paper was published. So that is extremely helpful. And therefore, by putting more citations in your paper, you make your paper more discoverable to more people, which is always a good thing.

2:14:00.2 SC: Sam Cox says: I wonder if the missing antimatter problem, the baryogenesis problem, is somehow connected to the mysteries of dark matter and dark energy. Perhaps the missing antimatter was sponged up by or morphed into dark matter, at an earlier moment in the Big Bang. Is it reasonable to expect there exists dark antimatter?

2:14:17.6 SC: Actually, I forgot. I’m going to group this with another question. Andrew Vernon Smith says: Can you explain whether or not it’s possible that the weak force may have been involved somehow in bringing about the universe, including time and space, via the Big Bang or otherwise, and if so, whether it’s possible that the right-handed matter and left-handed antimatter may have proceeded backward in time, since only left-handed matter and right-handed antimatter interact with the weak force in our forward-in-time reality.

2:14:46.4 SC: So the connection between these two questions is, matter and antimatter somehow being connected with bigger cosmological questions. And there is a paper… There was a paper that was written about a scenario similar to what Andrew is talking about, by Latham Boyle, Karen Finn and Neil Turok, a couple of years ago, about a CPT symmetric universe, where they suggested that in a way, morally similar to what Jennifer Chen and I suggested years ago, and Anthony Aguirre, former Mindscape guest, and Steven Gratton suggested around the same time, a back-to-back universe, where there was sort of a throat or a middle point, and then one part of the universe goes forward in time and the other goes in the opposite direction.

2:15:29.9 SC: Of course, both sides of the universe have people in them that call the direction toward the throat, the past. So it’s a two-sided time kind of scenario. The spin that Boyle and Finn and Turok put on it was exactly this kind of idea that there is a violation not only of time symmetry between the two halves, but CPT symmetry, which relates matter and antimatter and also right-handedness and left-handedness. So roughly speaking, sort of more matter on one side, more antimatter on the other side. I haven’t looked into that scenario very carefully, but you’re welcome to look it up. Just Google CPT symmetric universe, and you will find it.

2:16:10.3 SC: Relating that to Sam’s question, perhaps the missing antimatter was sponged up into dark matter in an early moment of the Big Bang. There are scenarios like that also. So especially, here is something that you can think about. The total amount of dark matter in our universe is around five times by mass as much ordinary matter in the universe. And that’s a small number. Five is not a very big number. In principle, these numbers are completely unconnected. The usual ways that we have of predicting the amount of dark matter and the amount of ordinary matter are just completely unrelated to each other. So why are they so close in density, a factor of five?

2:16:54.0 SC: Well, maybe, the theory goes, the dark matter particle is… And this is one version of the theory, there’s other versions of it. But one version is, maybe for every proton in the universe, there is one dark matter particle that weighs five times as much as a proton. And so maybe there’s some process that takes a baryon number and splits it, so that a positive baryon number goes into ordinary matter, a negative baryon number goes into antimatter, in some sense, and that’s why there’s more dark matter… Did I say antimatter? Negative baryon number goes into dark matter, is what I meant to say. And maybe that’s why there’s five times as much dark matter as ordinary matter.

2:17:35.3 SC: Those kind of scenarios don’t easily fit together. In theoretical physics, what you get to realize that some theories, some models, some frameworks that you propose, once you propose them, everything just fits. It just works and everything is great. Others, you kinda have to struggle to make them work, a little bit. And this is more of the latter, but that’s not to say that we just haven’t thought of the right way to do it. Finally, you say: Is it reasonable to expect there exists dark antimatter?

2:18:00.8 SC: Yeah, it’s perfectly reasonable. We don’t know for sure, one way or another, but if you think of all the different ways to have a relic abundance of dark matter, most of them predict that there is an equal amount of dark matter and dark antimatter, or that there’s no difference. Like photons, for example, don’t have an anti-particles, they interact with each other, they can be created or destroyed, but they’re bosons, so you can’t really distinguish they’re photons or neutral bosons. So there’s not a real difference between a photon and an anti-photon. That’s why you don’t talk about it.

2:18:35.1 SC: If the dark matter is a neutral boson, like the axion, for example, then there wouldn’t be dark antimatter ’cause there’s just no such thing as an anti-axion. On the other hand, if the dark matter is a fermion, which many WIMP models are, weakly interacting massive particles, then the theory is that, at some point, the dark matter and the dark antimatter were just annihilating with each other in the early universe, when it was very dense, but they were only interacting weakly with each other, so when the density goes low enough, they just don’t find each other. It becomes rare for a dark particle and a dark anti-particle to bump into each other, there’s no long range force between them, like electromagnetism. There’s only a very, very short range, weak nuclear force, so the two particles have to come very, very close together to actually annihilate. And that is the standard picture of weakly-interacting dark matter.

2:19:29.3 SC: So in the WIMP model, you expect there to be half dark matter, half dark antimatter or versions where, once again, there’s no difference between matter and antimatter in that sector, which is why you can still hope that in regions where you expect the density of dark matter to be very high, like at the center of a galaxy, maybe you still see some dark matter and dark antimatter annihilating into gamma rays and you can detect it that way. That was a big hope for the Fermi space telescope, to look for gamma rays from dark matter. The sad news about the Fermi space telescope is, they found a lot of gamma rays, but it’s really easy to make gamma rays, even if they have nothing to do with dark matter. So maybe some of them are due to dark matter, but we just don’t know. So far, we’ve been able to find less exotic explanations for all the gamma rays that we were able to observe.

2:20:19.7 SC: Okay, Tony B says: This question is inspired by the recent movie, Tenet, which engages with ideas on the entropic arrow of time more than many. You’ve spoken many times about how we remember the past because in that direction, our extrapolations are pinned down by the low entropy boundary condition at the beginning of the universe. If the low entropy boundary condition were in the other direction, we would remember in that direction the same way. Do you have any intuition as to what kinds of events could happen at the boundary between two regions of a universe, one region having a low entropy boundary condition in one direction, and the other having a low entropy boundary condition in the other?

2:20:56.1 SC: No, I don’t. I can give you a feeling for why it would be weird, but I don’t have a well-thought out model for exactly what things would be like. It would be weird because things would seem, to you, to start acting in ways that were anti-entropic, that were sort of despite your best efforts, lowering the entropy of the universe, which is just a weird thing, packs of cards spontaneously organizing themselves into order, two, three, four, five, six, etcetera, cream and coffee unmixing, glasses unbreaking, things like that.

2:21:34.2 SC: Craig Callender, who is a philosopher of physics, who thinks about these things, gave you a thought experiment, wrote down a thought experiment, where it’s like there’s a prediction, a low entropy boundary condition is precisely a precognition event, where you know what’s going to happen in the future and you can’t stop it. So the example he gave was, someone tells you that on a certain date, all the world’s Faberge eggs are going to assemble themselves into a certain drawer in a certain room, and you say, “Well, no, I’m going to stop it, I’m going to put up guards and prevent it from happening.” But all sorts of crazy things start going wrong, the guards are sick and someone drops the egg and it gets mistaken for something else, and eventually they end up there in that drawer, and you’re like, “How in the world is this happening?” And it would seem spooky and magical, it would seem like some intelligent force is working against you.

2:22:24.7 SC: Now, that’s not the whole story, because if you were truly at the boundary, then there’s one of two possibilities. Either, there could be living beings… Remember I said before, living beings require that entropy is increasing, they dissipate. So either there’d be two types of living beings which dissipated in opposite directions, which would be very weird, or more likely, it would just be impossible for there to be any living beings at all, near that boundary. But again, I don’t think anyone’s really worked out the details of what that would be like.

2:22:56.3 SC: Jeff Babon says: I’m trying to understand how much of the twin paradox is due to the fact that one twin accelerates while the other doesn’t. If I have a circular track, one light year in radius and I’m traveling around the track near the speed of light, I will feel a centripetal acceleration. If this track is next to Earth, then as I pass it, my twin brother and I will synchronize our watches. By the time I come around again, I think I will have aged more slowly to my twin, even though we’ve both been feeling acceleration, approximately 1 g the entire time. If that’s true, what makes my frame of reference special as it is because… Is it because only I have moved relative to the universe as a whole?

2:23:34.4 SC: So no, it is not because you’ve moved relative to the universe as a whole, and I’ve said this before, but I’ll try to say it again. I would just forget about words like frames of reference, or acceleration. Or any of these things. None of this is the point. The time dilation in special relativity is exactly the same as saying that if I have two paths between two points in space, those two paths can be different lengths. That’s it, that’s all that there is to it. If I put two dots on a piece of paper and ask two people to connect them with a curve, one of those curves is going to be shorter than the other, and if one curve is as short as possible, it will be a perfectly straight line, any non-straight line will be longer. And that’s exactly the same thing that happens with the twin paradox, except in time rather than in space, any non-straight line is shorter in time, experiences less time. So it’s not that it has to do with acceleration. The point of the twin paradox is not acceleration, the point is the length of your trajectory in spacetime.

2:24:42.3 SC: Of course, the rule is that, the longest duration path has no acceleration, so therefore, if you do accelerate, then you’re going to experience less time, okay, so that’s where acceleration comes in, but it’s not because it’s primary, it’s not because the acceleration is causing the aging in any sense. So Tim Maudlin, actually, we mentioned books about philosophy of science, so Tim Maudlin, in one of his philosophy of science books, has a very good illustration of this point. Think about the usual twin paradox set up, where one person just stays home, forget about the acceleration you feel on Earth, that’s a complication that we don’t need to worry about. One zooms out on a rocket ship and then accelerates and zooms back. So there’s sort of a triangle that you draw in spacetime in your mind, between the long edge being the stationary twin and then the two shorter edges with a bend in between, being the twin that goes out on the rocket.

2:25:40.5 SC: So you might think that it’s the acceleration at that turnaround point, that is doing the work. But Tim says, well, imagine the same amount of acceleration, but along a different path, namely, the first path was accelerate a lot outward, but then coast, and then turn around, so you accelerate again, and then coast, and then accelerate finally, to land at the same point you started with. So three bouts of acceleration. Just put those three bouts of acceleration much closer to each other. So accelerate out, immediately accelerate back and then accelerate to stop, so you get a much tinier triangle and then you sit there along the same path.

2:26:20.4 SC: In this case, you’ve done the same amount of acceleration as the first version of the experiment, where you went way out and came way back. You just did it all quicker, you didn’t have the long coasting periods in between. But the difference in time experienced by the two twins is much smaller in that second difference, in that second example, because the length of the path is much closer than… Much closer to the original stationary twin. So the acceleration allows you to have a different length path through spacetime, but it’s not the point. It’s not the causing of the difference in time duration. It’s the length of the path that is the cause of the time duration being different.

2:27:00.4 SC: Okay, Jeff B says: You’ve stated that you are against the idea of teleology underlying physics; however, it seems as if this would be difficult to judge. After all, the laws of physics may be progressing towards some final teleological state, but the path that it takes is simply too complex for us to notice or understand. In fact, this seems logical, based on the fact that matter tends toward the lowest energy state. This is reminiscent of the ancient notion of elements wanting to rise or fall. What do you make of this?

2:27:26.9 SC: So I’m not against teleology underlying physics, I just don’t think it’s there. The laws of physics. As we currently understand them, have no teleology in them, and there is no problem that we are faced with, under current knowledge of the physical world, that is solved by teleology. So I just think it was an old idea that we can get rid of. You mentioned the idea that matter tends towards lowest energy state, but of course that’s not true, because energy is conserved, at least to the classical limit. What you mean when you say matter tends toward the lowest energy state is that, if you have a dissipation full system, if you have a system with friction, then the system will, in fact, go to the lowest energy state, but what it’s doing is increasing the entropy of the universe.

2:28:13.7 SC: And that is not because there is some future high entropy boundary condition, it’s because high entropy is natural. So the thing is that, ordinary dissipation full systems tumbling down to their lowest energy state, doesn’t require any future condition at all, it’s just the natural thing to expect to happen, just as entropy is increasing is natural. So there’s no reason for teleology and there’s no evidence for it. That’s why I don’t think it’s an interesting thing to think about. That’s not to say that I would mind if we got new information that changed our minds.

2:28:47.5 SC: George Atenasoff says: Are you concerned about carbon emissions? We’ve begun using electric car transportation. Battery manufacturing, though, is very carbon-intensive and the subsequent battery recycling or disposal will create a nightmare 20 years from now. And there’s more details in there.

2:29:03.8 SC: So roughly speaking, yes, I am concerned about carbon emission, but you can’t… The various arguments you put forward in the question, are all words, and that’s not enough. Just exactly as I said with other questions earlier, sometimes you gotta do the math. So on the one hand, burning fossil fuels emits carbon, fossil fuel greenhouse gases into the atmosphere; on the other hand, various parts of the electric vehicle cycle also emit greenhouse gases into the air. But you can’t just say, “Well, they both are bad, therefore they’re equal.” You gotta measure it. You’ve got to actually do the calculation. So people have done the calculation, counting not only the power that goes to actually charge the batteries of the electric car, but also the mining and the manufacturing process and the waste process and so forth.

2:29:54.9 SC: And roughly speaking, to the best of my understanding, the answer is an electric car gives off about one-third of the carbon emissions of an equivalent internal combustion engine car, so it’s way less, roughly speaking. So it’s a kick in the right direction. It will be much better if we can actually get that electricity from solar power or something like that, rather than burning coal for it. But my actual answer to the question is, you have to do the math. And so far, the math that’s been done indicates electric vehicles are much better for the planet than fossil fuel-burning vehicles are.

2:30:32.4 SC: Chris says: Most of your discussions of philosophy that I’ve heard seem to be within a Western framework. What are your thoughts on others, such as Eastern philosophy? This relates to the topic of meta-philosophy. Do you have any easily summarizable views on that?

2:30:46.6 SC: I don’t think I have any easily summarizable views on that. My view about philosophy is, I care about what’s right and wrong. I’m interested in the history of philosophy, to the extent that it’s cool and interesting, but to the practical extent of trying to understand the world, I don’t care whether a good idea comes from Eastern or Western or whatever. And typically, the best ideas are relatively recent, not because recent people are smarter, but because they’re building on ideas from a long time ago. So when I’m thinking about, what happened at the Big Bang or the quantum measurement problem, there’s no difference in my mind between Eastern and Western philosophy, wherever the good idea comes from, I care about.

2:31:28.0 SC: Having said that, of course you’re right, that when I’ve been discussing philosophy, my touchstones are Western philosophy, that’s what I’m trained in, to the extent that I’m trained at all. So that’s my bag of tricks, the knowledge base that I’m working from, not from any conscious disparagement of Eastern philosophy. So clearly, what I should do is have a podcast episode about Eastern philosophy. Let’s see if that happens soon. Stay tuned for that.

2:32:00.2 SC: Okay, two more questions, I’m grouping together. Sandra Stookie says: In the podcast with Elizabeth Strychalski… There’s a C in her name that doesn’t get pronounced, Strychalski. The idea that cells are information processors and climb entropy gradients came up again. How is the idea that physical, biological computational systems process information and make choices or use strategies to optimize certain outcomes compatible with a purely deterministic evolution of the universe? In what sense can an organism or a self-replicating piece of chemistry influence its future? That kind of presupposes that there were alternative futures to begin with, that information about the environment can truly be exploited.

2:32:42.1 SC: Then the other question, somewhat related, is from Jason, who says: You’ve said before, in answering questions about free will versus determinism, that as a compatibilist, you find the interesting question to be about how to operationalize that. So how do you operationalize that? When does it make sense to treat people as if they have free will and when does it not?

2:33:01.4 SC: So the point is… So I can’t give a very good answer to Jason’s question. I think it’s a hard question. A good question. It requires more work. And I’m not an expert, but I’ll talk about it a little bit. To Sandra’s question, I can be a little bit more specific. The question, in what sense, can an organism or self-replicating piece of chemistry influence its future? This presupposes there are alternative futures to begin with. Well, there are alternative futures, as far as the person knows or as a self-replicating piece of chemistry knows.

2:33:34.5 SC: I think, if I’m a person or a bacterium or whatever, in this day and age, I happen to know that I’m made of particles or quantum wave function, and there are some laws of physics underlying it. But what if I didn’t know that? That doesn’t change my everyday life very much. If I didn’t know that, what would… How would I act? I don’t know what’s going to happen in the future, and you can say, yes, but the Laplace’s Demon knows. I’m not Laplace’s Demon, I’m not friends with Laplace’s Demon, I can’t get any information from the Laplace’s Demon about this. To every single… In every single sense that is relevant to me, thinking about the future, there are a whole bunch of possible alternative futures, and my choices will help bring them about.

2:34:19.5 SC: As I’ve said before in talking about free will, literally every human being acts that way. There are no human beings that truly act like they don’t make choices about the future. There are people who… Certainly, if you’re leaving comments on Patreon, you’re trying to either learn something or influence something, you’re trying to have a causal effect on the future. That’s why you’re doing it. There’s simply no coherent way to talk about human action without opening, without involving the idea that we have a causal influence on the future due to the choices that we make.

2:34:52.5 SC: So that’s the easy part of the question. What Jason is saying is, “Okay, there are conditions when we don’t have a causal impact on the future,” like when you’re asleep, [chuckle] when you’re unconscious, then you’re not making any choices about the future. Maybe your brain is turning, but you’re not operationalizing it, as you say. In other cases, if you have lost capacity somehow, if you’ve lost mental capacity or even physical capacity, there’s famous cases where you have a brain tumor, arguably, when you’re addicted to something, you just have no choice in some sense. So then the question is, how do you really draw the line between we should treat you as having had a choice and we should treat you as not having had a choice? I think it’s a good question. I think that there are easy cases, there are easy edge cases. If you’re asleep, you’re not making choices. If you’re awake and competent and deciding what shirt to wear that day, you’re making a choice.

2:35:48.8 SC: But there are fuzzy cases in between, and I think that the lesson of that is that we should think hard about what exactly should be done at that boundary where there are fuzzy cases. The lesson should not be because the boundary is fuzzy, the boundary doesn’t exist. Or, because the boundary is fuzzy, we should treat everything as if it’s either deterministic or indeterministic. Sometimes you gotta live with the fuzziness, sometimes you have to live with the idea that these human ideas, these coarse-grained, macroscopic emergent phenomena are fuzzy. That’s okay. There’s many things that are fuzzy, I think that right and wrong are fuzzy, so I don’t think it’s an argument against taking it seriously, so sorry for not giving you a once and for all answer, Jason, but I don’t have a once and for all answer, even though I’m convinced we should be looking for the best answers that we can invent.

2:36:38.1 SC: Preston Justice says: You’ve been successful dabbling in philosophy, therefore, is it logical or realistic to assume that one could go counterclockwise and find their way into contributing to the sciences without shutting up and calculating from the field of philosophy of science, physics in the humanities department?

2:36:54.8 SC: Sure, it’s absolutely possible but it’s work. It’s not easy. So, the word dabbling is the wrong word to use. You have to do the work, you have to… In particular, you have to take seriously both the knowledge base and the expertise of people who already do this thing, so that work is less visible if you’re moving from science to philosophy, ’cause if you want to become knowledgeable about what’s going on in philosophy, you read or you talk to people, whereas if you want to become knowledgeable about what’s going on in science, you might need to have a laboratory or something like that. This can happen. We had an example not that long ago. Ann-Sophie Barwich, who talked about the philosophy of smell, she trained as a philosopher and now she has a lab where she studies the sense of smell in the lab. So you can absolutely do that. There’s other people who think about consciousness, etcetera, who are philosophically trained and then move into directly experimental science.

2:37:58.1 SC: For physics, yeah, sure. I mean, for people who are philosophers of physics with a good background in physics, many of them do things that are just indistinguishable from doing physics, and you could even imagine falling in love with a particular physics problem that had no more connection with philosophy. But again, you just have to have respect for the existing knowledge base in those fields, then you can do whatever you want. That would be my attitude.

2:38:26.5 SC: David Wright says: The latest podcast with Elizabeth Strychalski on synthetic cells was fascinating and touched on some of the same exciting questions about the boundary between physics, chemistry and biology that Michael Levin is dealing with in his work on morphogenesis at Tufts. You ended up discussing whether living systems actually require a cell membrane, and if it was possible there was a more general synthetic form of biological systems. This reminded me of Karl Friston, who has identified a model for biological systems based on his free energy principle, another previous Mindscape guest.

2:38:58.4 SC: The model identifies internal states, external states, active states and sensory states that operate to minimize the difference between a known internal state and a Bayesian predicted external state. A cell membrane may simply be an evolved way to limit the computational bandwidth needed to process the model algorithm by reducing the number of internal state variables. What the cell model and non-cell model share in common is that they would both be based on Markov blankets with different scales and numbers of states. So the question is do you think Dr. Strychalski should consider Friston’s model in her research?

2:39:29.7 SC: So I’m not going to give Elizabeth any advice about what to do for her research, she knows much better than I do what the relevant things that could be helpful are, but I do think that Friston’s model is very interesting and worth taking seriously, let me put it that way. What I don’t know is whether it’s… So the way I think about it is the following. When we look at real living beings, cells are the basic building block, and cells are not just collections of atoms and molecules, they’re a specific kind of collection of atoms and molecules that are compartmentalized in a very clear way. There’s a lipid bilayer that surrounds the cell and acts as the cell wall, and there’s an inside and an outside, and there is an internal structure inside that grows through evolutionary history and so forth.

2:40:15.4 SC: So the question is is this just an accident? Could it have been completely otherwise? Could there be no cell walls? Is it convenient? Is it useful to have a cell wall, but not necessary, or is it absolutely necessary? Is there no other way to make life? One of the things that Elizabeth does in her research is non-cellular synthetic biology, so she studies biological processes in artificial environments that are not surrounded by cell walls at all. But my point is that I don’t know the answer to this question of whether cell membranes are necessary, helpful, or completely accidental.

2:40:54.4 SC: I think that what Friston has done is giving good reason to believe it’s not just an accident, which I was very willing to believe anyway, so it doesn’t require much information as a good Bayesian. My prior was already up there, but what he’s saying is, “Here’s a way in a very abstract high-level sense of thinking about the usefulness, the role played by that cell membrane as a way of sort of limiting the bandwidth of information that comes back and forth between the external world and the internal world.” That makes perfect sense to me. There’s a huge danger that we already know the answer, and so we’re inventing a story after the fact.

2:41:33.8 SC: So I’m not sure that that is the only or even best way to think about the usefulness of cell membranes, maybe it is, but I think that this is stuff that my impression is it’s cutting edge stuff, that is to say there’s not settled answers that people agree on, that might just be because I don’t know the field very well, but it might be because biology is hard and we’re working on it. I certainly think that that kind of idea is very useful, very interesting, let’s put it that way, very worth following up on.

2:42:05.2 SC: Brendan says: I was interested in reading your textbook, Spacetime and Geometry, but I was wondering how much prior knowledge in mathematics and physics is roughly required. I have all of your other books and I followed along to the Biggest Ideas in the Universe, but was unsure whether that would still be enough without a rigorous understanding of the underlying math and physics. My background is in computer science, and it’s been over 10 years since I took a differential equations class.

2:42:30.3 SC: Well, it’s always hard to say. The goal of the book is to be self-contained, so it’s not assuming that you have a lot of prerequisites, [chuckle] but if you’re not used to thinking in certain ways, the material can be very, very intimidating. It’s more a matter of is the material sufficiently similar to things that you’re used to seeing, that it’s not too scary, or do you say like I have no idea what all these symbols mean?

2:42:58.7 SC: It’s not a matter of explicit prior knowledge, so much as a matter of being ready to handle this kind of thing. The one counterexample of that is you do need to be pretty comfortable with differential equations, in particular, partial differential equations. You have to be able to know what a partial differential equation is, it’d be very, very helpful if you were used to seeing solutions to the wave equation and things like that, because we’re going to very quickly use that and that new notation that you haven’t used before, and if you’re grasping both what the equations are and the new notation, that’s much harder than just looking at the new notation.

2:43:38.9 SC: So the single thing that is probably the shortest distance between very basic background knowledge and that GR book is actually something like a good E&M book, electricity and magnetism, a good E&M book being one that uses special relativity in a very central way, like you can do E&M, even though electricity and magnetism a la Maxwell was the inspiration for special relativity, it was invented before special relativity, so you can certainly learn electricity and magnetism without putting it into that special relativity context, but it can be put into the special relativity context. And once you do that, your transition to general relativity will be much, much quicker and easier, so that’s what I would say.

2:44:25.7 SC: I’m trying to think of an example of a good book. I’m not super familiar with the textbook landscape on electricity and magnetism. I believe there is a book by Ohanian, Hans Ohanian, that I liked and really used special relativity in a central way, so that might be a good warm-up.

2:44:45.4 SC: Richard Graf says: Try as I might, I can’t grasp the connection between symmetry and conservation laws as worked out in Noether’s theorem. Can you explain the relationship in relatively non-technical terms or is this one of those somethings deeply hidden that requires an understanding of the math to comprehend?

2:45:03.2 SC: Yeah, it’s a good question. I did want to cover Noether’s theorem when I did the Biggest Ideas in the Universe videos, and I ended up doing it. It is in there in the video on symmetry, I talk about Noether’s theorem, and I give a little argument in favor of it. Now, it requires work, because you first need to understand the Lagrangian formulation of classical mechanics. Once you understand Lagrangians, you can then understand a little bit about symmetry and then you can understand Noether’s theorem.

2:45:32.5 SC: So there’s a bit of technical background required one way or the other, and… Oh, I should be honest… Roughly speaking, got that explanation of Noether’s Theorem that I gave from Feynman, from the Feynman Lectures on Physics or something like that. I believe that was the book where it was from, and to be honest, it’s not intuitive, really, like when I say intuitive, it depends on what your intuition is. Sometimes you’ve been using Lagrangians long enough in your life that things become intuitive that weren’t intuitive before you use Lagrangians, but it’s definitely not something where you could hand-wave to a person on the street and get them to see not just what Noether’s theorem says, but the proof for why it’s true.

2:46:13.1 SC: I don’t know a high school level way of saying how the proof for Noether’s theorem actually works out. It would be interesting to get that, but you know, it might not be possible because Noether’s theorem only applies to classical systems for which there exists a Lagrangian description, okay. There are systems for which Noether’s theorem doesn’t apply, so I suspect that you need to know what Lagrangian is to get there. That’s my suspicion.

2:46:42.5 SC: Rebecca Leshua says, or Leshua: Do you agree with Max Tegmark’s basic premise that the universe is essentially mathematical in nature? Certainly, a complete theory of everything would be written in mathematics, but is there something that breathes fire into the equations? Or is the universe just the equations or somehow isomorphic to them?

2:47:00.6 SC: So I guess I alluded to this earlier, but I think that last parenthetical statement, phrase that you’re using is really, really crucial there, because I do think the universe is isomorphic to some set of equations. I do not think that that’s more or less the same as saying the universe just is the equations. The universe is the universe, the universe is sui generis, the universe is the unique universal thing. I’m a monist in that sense.

2:47:27.8 SC: Again, I could be talked out of it by evidence or reasoning or whatever, but as far as I can tell, there’s something called the universe, and it is the thing. To try to explain what it is in other more primitive terms is a fool’s errand. There are no more primitive terms to use, it’s what exists, but we can describe it, we can describe it using equations, using laws of physics, and there’s nothing that breathes fire into them, they’re just a language that we use to describe the universe.

2:47:54.9 SC: So the universe is mathematical in the sense that we use math to describe it, but it’s not made of mathematics or anything like that. As I said, there are different versions of mathematics, what would it mean for the universe to be made of mathematics? There are parts of the universe that behave in ways for which mathematics is a useful way of talking about them, that’s how I would say it.

2:48:18.2 SC: Anonymous says: I’ve heard physicists say that time and space switch roles inside of a black hole. I’ve also heard them say that the whole universe is inside a black hole. Are these statements true in some sense?

2:48:28.1 SC: No, [chuckle] neither one of those statements is true in any sense. Sorry about that. Let’s go to the first one, time and space switch roles inside of a black hole, just false, just totally wrong. What would that even mean? I don’t know what that would mean. I know why they say it, I know what mistake they’re making, but it is totally a mistake. The mistake they’re making is the following. When we do physics in spacetime, it is very often to put coordinates on spacetime to find things. So to be able to say where something is and when something is happening, we put coordinates on space and coordinate on time. So let’s imagine that we do that in a way that is naturally adapted to a black hole, let’s say a simple non-rotating Schwarzschild black hole, okay?

2:49:14.4 SC: There’s a very obvious version of polar coordinates, spherical coordinates that works for a black hole, and you have time, you have space, they both have certain distances in both time and space that are described by the spacetime metric, okay? Now let’s say, and for all our purposes here, these coordinates are T for time, R for the distance, the radial distance, and then there’s theta and phi for the angles around some access that you pick, and who cares about theta and phi. It’s a spherically symmetric black hole, so really what we care about are T, the time coordinate and R, the distance, the radial coordinate away from the black hole.

2:49:56.0 SC: So here’s a fun fact, if you naively take those coordinates that you set up outside the black hole and extend them into the black hole, then what you were calling R, the radial coordinate, becomes timelike, that is to say increasing your value of R moves you in a timelike direction, not a spacelike direction, and T, what you were calling the timelike coordinate, becomes spacelike inside the black hole.

2:50:28.0 SC: That’s a true statement, but hopefully, you can see the vast gulf [chuckle] in between that true statement and the very false statement that time and space have switched roles. Time and space didn’t switch roles at all. We just chose a dopey coordinate system, that’s all. Don’t reify your coordinate systems, don’t take your coordinate systems too seriously. If you fell in to a black hole, it was a big black hole, so you weren’t being spaghettified or anything like that, you wouldn’t even notice that you were in a black hole, nothing happens when you cross the event horizon. You would think that if time and space had switched their roles, you would notice, [chuckle] but in fact, you don’t. You wouldn’t be using coordinates when you were just floating around, you would just be falling in and waiting ’til you hit the singularity.

2:51:16.3 SC: The other true statement is that the singularity is not to your left, if you’re in a black hole, it’s to your future, okay? It is timelike separated from you. But again, it always was, and it never was toward the center. Whenever you were told the singularity was at the center of a black hole, they were just lying to you, you’ve been lied to a lot, I’m sorry to tell you that, but the singularity is in your future. It’s kind of like a big crunch.

2:51:39.7 SC: So the other half, the whole universe is inside a black hole. No, clearly not. Because if that were true, if we were inside a black hole, then guess what, there would be a singularity in our future, which we would inevitably hit. There’s no reason to think that there’s a singularity in our future that we will inevitably hit, in fact, the simplest version of cosmology says the universe will just continue to expand and accelerate and empty out forever, that doesn’t sound very much like a black hole at all. But again, there is a very dumb argument that could lead you to believe that it is, namely, the following. If you have enough matter and you collect it into a region of space and calculate its density and its size, there is a conjecture that if you have enough matter in a region, it will have to collapse to a black hole, okay, and if you take a big enough part of the universe, it turns out we do have that much matter, so therefore, shouldn’t you be collapsing to a black hole?

2:52:37.9 SC: Well, they lied to you, again, because it’s not true that if you get enough matter, you have to collapse to a black hole. It’s true that if you get enough matter, you either collapse to a singularity in the future or you came out of a singularity in the past. Guess what? [chuckle] We came out of a singularity in the past, which we call the Big Bang. If anything., the universe is like a white hole, a white hole is just a time reversal of a black hole, which starts in the singularity and spits things out. It’s not exactly like a white hole either, because there’s no event horizon around it as far as we know, but it’s much more like a white hole than it is a black hole. The universe is nothing like a black hole, roughly speaking.

2:53:21.7 SC: Ashik Dragneel says: What is the universality of computation? Are humans universal, too? Can humans understand anything and everything in the universe given in our limited perception of the world?

2:53:36.5 SC: Well, this is a subtle, complicated question, and also sort of at the periphery of my own knowledge, so maybe someone who knows better can chime in. There is an idea called the Church-Turing thesis. So you can go Google Church-Turing thesis, these are two people’s names, it’s nothing to do with the church, [chuckle] okay, Alonzo Church and Alan Turing. And the thesis is, these are mathematicians, okay, so they’re going to phrase things precisely, not vaguely like we physicists would phrase them. Consider computable functions, so you have some, in fact, make your life easy, make things finite, so rather than functions of real numbers, think of functions of integers, okay. So there’s some function from integers to other integers, and some of these are calculatable, computable, and some might not be, like you can invent a non-computable function, and what the Church-Turing thesis says is that if your function is computable, then it can be computed by a Turing machine.

2:54:36.3 SC: A Turing machine is just a computer. Let’s call it a computer to make it simple. Now, the Church-Turing thesis is not proven as far as I know, it’s not something that is absolutely established, but people think it’s basically true. Okay. Maybe it has been proven. I honestly don’t know, like I said, it’s not in my wheelhouse, but people treat it as true. So what they’re saying is that this class of computable functions, if it’s computable in one way a computer can calculate it. That’s a little bit far away from the phrase as you put it, can humans understand anything and everything in the universe.

2:55:13.6 SC: Well, if we say, number one, that the human brain is a Turing machine, if it has the capabilities of a Turing machine, which I think is fair. It might not be a very accurate Turing machine. We make mistakes, unlike a good computer, but we have the computational capacity to be Turing-like. But then in principle, given time and enough graph paper, we can calculate all these functions, that is not the same as saying we can understand anything and everything in the universe.

2:55:40.5 SC: I don’t even know what that means, understand anything and everything in the universe. So something like discovering the correct laws of physics might not be equivalent to computing a function. So there is sort of the rigorous statement of the thesis, and then there is the sort of informal understanding, and the informal understanding is just not that on that firm ground. The informal understanding is once you can compute things symbolically like a computer, then you can compute whatever you want, and all of the effort is… All the work is being done by the phrase “whatever you want.” So I would be a little bit careful. Let’s put it that way.

2:56:22.5 SC: I do think that… I want to have it both ways. I want to have my cake and eat it too. On the one hand, I think that the ability to do symbolic computations like a Turing machine, like a computer, is something that humans have and something that earthworms do not. I think that there is some kind of phase transition in our thinking ability that happened along the evolutionary timeline, so that human beings can symbolically manipulate things, can talk to each other using language and words in ways that at least some other animals or plants cannot and that gives us an ability to calculate things that is above that of earthworms. But I don’t want to stretch that to anything and everything ’cause I don’t know what exactly that means. Maybe, I just don’t know. Let’s put it that way.

2:57:07.0 SC: Okay, Varun Narasimhachar says: Why do you think our understanding of quantum mechanics is incomplete? I feel we understand it operationally just as well as we do classical mechanics, but seem to insist on fitting it to our classical intuition by adding unnecessary features beyond what’s operationally accessible. As for questions like what really exists, I think we had no meaningful answers in classical mechanics either, e.g., does mass exist? Do electric fields exist?

2:57:35.5 SC: I think it’s pretty straightforward why our understanding of quantum mechanics is incomplete. If you look at the textbook explication of quantum mechanics, you’re told things like when a quantum system is measured, its wave function collapses and the collapse is unpredictable, it obeys the Born rule, the probability is given by the wave function squared, etcetera, okay. You’re not told what it means to measure a quantum system, that’s just left incomplete. I’m not saying it’s not completable, but you’re just not told what counts as a measurement, does a single photon bumping into something qualify as a measurement? Do you need to be conscious to do a measurement? Could a video camera do it? How quickly does it happen? When exactly does it happen doing a measurement?

2:58:22.3 SC: Now, of course, you can try to do better, what I’m saying is the textbook version of quantum mechanics just leaves these questions unanswered. So that is clearly incomplete, there’s just zero question, zero controversy about the fact that textbook quantum mechanics is not a complete rigorous physical theory.

2:58:40.1 SC: You can try to fix it up, you can say, “Well, there’s decoherence,” blah, blah, blah, but then what you discover is that there are different ways to fix it up, and people don’t agree on what the right way is, that’s why you have Bohmian mechanics and many worlds, etcetera. The very fact that some people believe Bohmian mechanics and some other people believe many worlds, and some other people believe objective collapse models, and we don’t know which one is right is right there a proof that we don’t understand quantum mechanics, that our understanding is incomplete.

2:59:08.8 SC: If we did have a complete understanding, we would know which of those is right, if any of them, okay. As for questions like, what really exists. Your two examples are, does mass exist and do electric fields exist? Mass is not a thing. It’s a property. Mass, energy, momentum, these are properties that other things have, like electrons or electromagnetic fields or whatever. So mass, you can take it or leave it, there’s different ways of describing the same physical thing with using that property or not, but electric fields, do they exist? The answer is yes. [chuckle] In classical electromagnetism, that’s a very meaningful answer. Yes, the electric field exists. I think that if you give up on the idea of saying what exists, then you give up on the goal of understanding the universe, which is both a very plausible goal and one we’re making great progress towards. So that’s not a goal I’m going to give up on any time soon.

3:00:02.6 SC: Kathy Sieger says: In your recent paper, Consciousness and the Laws of Physics, you elaborated on the core theory, you’ve also made it clear what its domain of applicability is. Since Frank Wilczek proposed the core theory in 2015, and you endorsed it ever since, what’s your impression, did it gain more attention and approval in the physics community over time, and does Frank Wilczek approve of the way you constructed the core theory equation? Frank Wilczek of course, yet another previous Mindscape guest.

3:00:29.9 SC: So let me be clear about this, ’cause I don’t want to give too much credit to either me or to Frank Wilczek. What Frank Wilczek put forward in 2015 was the name, the core theory, [chuckle] the core theory itself was already there. It just didn’t have a name. And the core theory is the combination of the standard model of particle physics and general relativity thought of as an effective field theory in the weak field limit. Okay. So people knew before 2015 that that was a perfectly sensible theory to talk about. Frank’s point was, “This is what you need to understand the laws of physics underlying everyday life.” And so he gave it a name, the core theory.

3:01:10.3 SC: No one person invented it, it was put together by many different people, including Einstein, because general relativity came from him, and including Frank Wilczek, because the strong interactions, you need to understand asymptotic freedom to understand them, which was understood first by Gross, Politzer and Wilczek, and they won the Nobel Prize for that. So part of the core theory really is Frank’s, to his, Frank’s credit, but there’s also the weak interactions, the electroweak theory, for whom the most important person was Steven Weinberg, who sadly just passed away, but again, many other people, Murray Gell-Mann, Richard Feynman, arguably James Clerk Maxwell gets credit, ’cause he wrote down the electromagnetic part of it, etcetera. Dirac obviously gets credit, etcetera, etcetera.

3:01:53.3 SC: Many, many, many people contributed. Higgs, Englert and Higgs and Brout, so that was already there. So I don’t think that the physics community cares, they knew it already, the existence of a new label for it, doesn’t change their minds very much, so if you… And I’ve done this experiment, I think maybe what you’re really getting at is, if you go up to someone who is a working physicist who is an expert on quantum field theory, who really knows what you mean when you talk about effective field theories in the standard model, etcetera, and you say, “What do you think about the claim that if you take general relativity and the standard model and combine them into an effective field theory that qualifies as the laws of physics underlying everyday life.” And as we learn more and discover new things in physics, they won’t have any direct impact on everyday life, people will generally think about it for 10 to 15 seconds and go, “Yeah, that sounds right.” [chuckle]

3:02:52.9 SC: So they haven’t previously thought about it, generally, but they don’t object to it either. You asked, “Does Frank Wilczek approve of the way I constructed the core theory equation.” Yes, but again, I didn’t do anything. It was already there, it was just writing things down, the people already had thought about.

3:03:11.6 SC: Samuel Benjamin says: You’ve mentioned many times that the words we use in modern physics are poor descriptors of the phenomena we observe, wavefunction being a particularly bad one, although I still have to think hard about the various uses of the word space. Have you ever come up with alternatives for some of the more challenging terms and phrases that you feel do a better job at helping others understand the concepts, do you know of any other languages that describe these concepts better than English?

3:03:36.3 SC: So. I have in different moments, but not very seriously, like one I talked about, I think I talked about on the podcast not too long ago was smooth tension as an alternative to dark energy. But you know what? It’s just a joke. I wasn’t serious, and the reason why I was not serious is because you can’t really fight the battle of making sure that all the words we use are not only labels, but also definitions. Words are labels. When you say quantum mechanics, you mean a certain theory, it is not necessary that the theory be the mechanics of quanta. [chuckle] When you say the theory of relativity, you mean a certain theory. It is not necessary that that theory really had anything to do with one thing being relative to another, it’s useful that they are, it’s helpful, but really is a label, as long as you know what you mean, that’s what matters.

3:04:30.7 SC: And maybe we would like to imagine a perfect world of language and representation where all words really gave you an instantly correct idea what they’re referring to, but that’s not our world, that’s not the world we live in, and the battle to clean up the language so that things are more accurate is very, very hard to win, and maybe there’s certain sort of social justice examples where it’s worth fighting that battle, but for labeling physics theories, it almost never is. And I don’t think that any other languages do any better, I’m not really fluent enough in enough other languages to know, usually a language will just borrow the name from whatever language the original concept was invented in, so the languages tend to share the same terminology for these kinds of things.

3:05:21.3 SC: Okay, last question comes from David DeClut, who says: How do you decide when to end a podcast episode? See what I’m doing there, ending this podcast episode with this question? The question is: Ow do you decide when to end a podcast episode? Are guests ever caught by surprise when you suddenly say thanks for being on the Mindscape podcast, or do you prepare them in some way?

3:05:42.5 SC: Nope. They’re never [chuckle] prepared. Maybe they’re caught by surprise, I don’t know. I think it’s usually pretty clear that we’re winding down, maybe I’m wrong about that, I don’t know, but I’ve been on other podcasts myself more than enough times to be on the other side of the microphone and… Yeah, usually you get a feeling, very often, as you’ll hear in the podcast, I would give little clues that we’re winding down. I won’t usually say, “This is… ” Sorry, I shouldn’t say that, sometimes I say this is the last question, but what I realize is it’s often not true, so I try not to say that. What I will try to say is something like coming to the end or the last topic. The last thing I wanted to talk about is… ‘Cause there’s very often follow-up questions, right?

3:06:33.1 SC: So sometimes I do that, other times there’s a set of things that are natural to talk about and we’re coming to the end of them. Other times, there have been like a tiny number of times when the guest has really wanted to keep talking, [chuckle] usually like an hour-plus podcast is a long time and I’m talking to busy people, some people like to talk forever. Stephen Wolfram would have kept talking, but it was past 10:00 PM my time and past midnight his time when we were doing the podcast, [chuckle] so I don’t have the energy to keep going after two-and-a-half hours. And most people, like once you hit an hour there is something natural about an hour, even though it’s an arbitrary human invention, we’re trained or socialized to do things for an hour, TV shows, movies, whatever, hour, hour-and-a-half makes sense to us.

3:07:19.6 SC: After that seminars… After that, you begin to fade, so I think people are ready to wind things up. So I’ll give you two examples of counter examples, one is Jeffrey West, one of my first podcast guests, you can go listen… It’s like in the first 10 podcasts I did, Jeffrey West, it might be like five or six, and Jeffrey is great, and one of the things that is great about him is he can talk forever, [chuckle] and so… Just get him talking. And I told him, I was new, I was naive, and I said, “Look, we have plenty of time. We have over an hour,” but the mistake was, that was when I was visiting the Santa Fe Institute, where Jeffrey is a professor, and I was scheduled to be at a public event in Santa Fe at a certain time later that evening, and I literally had to leave.

3:08:11.7 SC: So I had to wrap it up, we had gone over for over an hour and Jeffrey was like, “I didn’t even get to talk about sustainability, I want to talk about that.” Like, “Well, okay, I gotta go, I gotta be on stage.” So that was one example. The other example was Kip Thorne, who said, “I thought you were going to ask me about what it is like to win the Nobel Prize, and I had a good answer all set for that.” And I said, “Yeah, I don’t care what it’s like to win the Nobel Prize. I care about black holes and gravitational waves and time travel.” But he said, it’s the one time when someone said, “Well, could you ask it to me now you’re done… Can you ask it to me now and edit it in,” so I did that, so I don’t know if it’s noticeable, but in that podcast, there was a time in the middle where we’re talking about the Nobel Prize, which actually was recorded at the end, and then I edited in.

3:09:00.9 SC: And it’s not just because… It’s not at all because Kip wanted to sort of bask in the glory of winning the Nobel Prize, he actually wanted to critique the whole process, he had thoughts about how it could be better and its shortcomings, and I always do ask if I remember, but… Which is almost always the case when I finish a podcast, finish the recording, I say, did we miss anything? I ask the guest. And usually they say, “We covered a lot. That’s good.” [chuckle] So I don’t think that we’re leaving a lot of things out. There are, certain people have a lot to say, certain people have a thing that they’re going to say in different ways over and over again, so different people are good, but most of the people who I’ve had on the podcast, I could talk for longer, and famously, there are other podcasts that go on longer than I do, but as I’ve said before, I am very gratified that the kind of people who I get on the podcast come on at all.

3:10:02.6 SC: I think I get really, really good people, that some people are even better than others, but the overall quality of people I get is very gratifying to me, I’m very happy with it. And they’re very busy people, and I don’t want to take up too much of their time. I’ve had people say, “Well, normally I say no to podcasts but for you I will say yes,” and that puts a lot of pressure on me to try to make it a good podcast and try not to waste their time in any way. So almost all the people I talk to have much more to say that is interesting than we get to cover in the podcast, but there is a limit to what is sort of appropriate.

3:10:35.6 SC: And with that, we’ve reached the end of this podcast, this AMA. Thanks, have a good month. Hopefully, our pandemic continues to evaporate and we can return to normality again, take care. Bye-bye.

[music][/accordion-item][/accordion]

8 thoughts on “AMA | August 2021”

  1. > So the right question to ask is, how can you change other quantities of nature and still have beings like us?

    You said you don’t know the answer to this. I just wanted to point to “The Anthropic Cosmological Principle” (by John Barrow and Frank Tipler) which/who you have probably heard of. The entire 738-page book is an attempt to answer this exact question, considering whether life is possible if any cosmological constant were changed. Not sure how up-to-date it’s been kept but it’s basically that.

    Love the podcast. All the best.

  2. Dimitris Papadimitriou

    About the Q/A at 2:48:
    The “switch” between radial and time coordinates that occurs inside the black hole event horizon is not just a coordinate phenomenon, without any fundamental significance (like e.g. the coordinate singularity at r=2m).
    The time translation Killing vector becomes spacelike inside the horizon, and this is directly related with the emission of Hawking radiation, and, also, with the fact that, inside the horizon nothing can remain static:
    Spacetime geometry is dynamical inside and r=constant hypersurfaces are spacelike, just like the future singularity that occurs inside, that, as you point out, is not “a point in space” but a future endpoint that acts like a “cut-off” for any worldline that crosses the horizon.
    For rotating BHs, things are more complicated, but (with the exception of the -unphysical anyway- extremal BHs that do not Hawking radiate and their r=constant hypersurfaces are timelike for r<m), things are otherwise similar, so this switch between the spacelike and the timelike character of the metric components that occurs does indeed maters!
    It's also connected with the growth of the interior of the (non extremal) BHs as the external time goes by:
    The coordinate time that, from the outside perspective, counts the age of the B. Hole, becomes, inside, the length of the 3-d hypertube, that grows until the end of the evaporation of the black hole (for spherically symmetric, non rotating or charged BHs.

  3. Dimitris Papadimitriou

    About the second half of this particular question: I agree, of course, that our universe does not resemble,at all, a black (or white) hole. Even the initial singularity of our universe, (if existed), is totally different from that of a white hole: It had zero (or close to zero) Weyl curvature, contrary to the divergent Weyl curvature of a white hole. I’m not sure, though, that the question was about the universe as a black (or white) hole.
    It seems (to me) that was referred to various speculations (from theories like LQG or Einstein-Cartan), that have to do with baby universes. Many theorists hypothesize that BHs have regular interiors, without singularities, so according to their models, spacetime is extended, somehow, into a new region, or universe.
    I’m not sure that these models have anything to do with our universe, if they can’t explain the initial low entropy and other properties that we observe, but it’s conceivable that such spacetime extensions may exist in quantum gravity and may resemble to some kind of universes after all.

  4. With regards to your answer to my question. I respectfully posed a question/idea that recently came to my mind. Quickly rummaging through my copies of ‘New Scientist’, a magazine you have contributed to, I noticed that the lead article to the 4th March 2017 edition was entitled ‘If the speed of light were infinite…’. In short it stated “You could achieve the same effect as inflation, however, if cosmic light speed started out infinite (or at least a lot larger) at the big bang and has been getting slower ever since as space has expanded.” It so happens that the idea I suggested is consistent with this if time commenced at a slower pace than it currently operates. [It seems to me quite plausible that time may have commenced at a slower pace than now given that matter as it exists now would have taken time to evolve and elementary particles, travelling at the speed of light, exist outside the dimension of time. Gravity’s attractive force would have slowed down such matter and surely would have introduced time in a gradual manner.] Further to this, I imagine gravity’s influence in the vast void of space between galaxies would be less than that within galaxies themselves and hence potentially alter the progression of time. After all, space/time is curved is it not? Why could it not progress more slowly when less influenced by gravity? Another ‘New Scientist’ a few months later (10th June, 2017) discusses the Higgs boson in relation to inflation matters whilst the 18th June, 2016 magazine’s lead article was ‘How to make Dark Energy disappear’, in which it is claimed “Dark energy is an illusion created by the machinery of our cosmological model.” There are other articles on these matters but I have other interests as our world disintegrates. My background is in weather forecasting and have I got stories there….. our failings in relation to climate change are inexcusable.
    In short, I cannot accept your claim “and so far that relationship fits all of the experimental data, it passes all the tests”. You can’t find 95% of what you are looking for as you discuss in your “The Great Courses” lecture series on ‘Dark Matter, Dark Energy’ which was produced in 2007. It is nearly 15 years further on, and no sign of it. The claim that the speed of light as we measure it now, our ‘arrival speed’, is a universal constant for ALL of time and ALL of space, allows us to interprete ALL of the information we are observing in a consistent manner. But if this is not the case, then we will have to get some aspects wrong. Don’t forget that Einstein didn’t get everything correct. Dan Hooper has a 12 lecture ‘The Great Courses’ series on ‘What Einstein got wrong’. He didn’t get everything right and as a weather forecaster who resigned 3 times in torment over the way the whole process failed due to political and communication failings, I know I didn’t get everything right. It was a humbling occupation.
    Your offering anybody, anywhere to ask you questions on anything, is a courageous undertaking. Certainly something I wouldn’t have the balls to do. There is only one of you, and many of ‘us’. Most of ‘us’ are stupid. Here in Australia, our arrogance over past issues with largely avoiding COVID is coming back to haunt us, as our collective embrace of the ‘Dunning/Kruger’ effect is coming to the fore. Having said that, there are some people with some sensible ideas worthy of further thought/reflection. I think I have asked you 3 questions. I agonised over all 3, and how best to pose them subject to your constraints. All 3 were quickly dismissed. I am not stupid. In fact I unfortunately have a rich history of intellectually intimidating people. Why? Because I don’t back down on ideas/principles that are important to me and I have a very good, though not quite perfect, record. It plays on their consciences. I will cease my contributions to your Patreon account. I haven’t earned an income for 13 years since I last quit my job and am living off my dwindling savings. I was sensible enough not to have any children as I knew life only had to get worse. I hope you will reflect on these comments and use them to re-focus the way you are operating your AMA idea. Some questions are worthy of further contemplation than you clearly give. Whenever I have resigned from a job things changed after I left and they were in keeping with what I maintained should have been done in the first place. I don’t expect any of these thoughts to find their way onto your web page given their nature. I took a screen capture of my comments regarding your recent “Arrow of Time” podcast as I thought there was a risk that they wouldn’t would have made it. Michelson/Morley couldn’t find the ether, but my comments did! Good-bye.

  5. Jesse Rimler asked about “a fundamentally different kind of musical harmony…a different kind of math”.
    We do have such a different kind of musical mathematics, based on Bessel functions.
    The familiar overtones of a 1-dimensional guitar string are described by the overtones of sine functions
    However, I understand that the overtones of a 2-dimensional drum surface are better described by Bessel functions, with a very different overtone structure.
    Percussion is your “different math”.

  6. If I were to have a dinner party with great physicists long departed,
    much of the fun would be asking them what they want to know about
    the knowledge we have now.
    After, maybe, feeding them a few teasers to prime the pump!
    It’d be curious what questions they have for the future.
    For that matter, what questions would YOU ask of future physicists?

  7. “I’m not friends with Laplace’s demon.” Can I please get that on a t-shirt or coffee mug?

Comments are closed.

Scroll to Top